You are on page 1of 67

Pain Medicine Board Review 1st Edition

Anna Woodbury Et Al.


Visit to download the full and correct content document:
https://ebookmass.com/product/pain-medicine-board-review-1st-edition-anna-woodbu
ry-et-al/
Expert I {QLiNE-l U L'l

Any screen.
Any tlme :- :::---—-* 4-671-1-

Anywhere. "9
Access additional resources online
at no extra charge by scratching
.IIIIIIII*
the pin code at the bottom right
and redeeming the code at
ebooks.elsevier.com

Unlock your additional resources today.


0 Visit ebooks.elsevier.com.

a Select English to continue.


9 Scratch off your code; enter your code in the
box and click next.
0 Follow the on-screen instructions. An email
will then be sent to you with final instructions
for accessing your online-only materials.
It’s that easy!

Place Peel Off


Sticker Ilnra

For technical assistance:


email expertconsult.help@elseviencom
ELSEVIER call +1-314-447-8200 (outside the US)
Use of the current edition of the electronic materials is subject to the terms of the nontransferable, limited license granted on expertconsultinltlingcom-
Access to the electronic materials is limited to the first individual who redeems the PIN, located on the inside cover of this book. at expertconsultinklingcom
and may.r not be transferred to another party by resale. lending. or other means.
2015M .t‘l
Pain Medicine
Board Review
This page intentionally left blank
Pain Medicine
Board Review

Anna Woodbury, MD
Department ofAnesthesiology
Emory University School ofMedicine
Veterans Afiairs Medical Center
Atlanta, Georgia

Boris Spektor, MD
Department ofAnesthesiology
Emory University School ofMedicine
Atlanta, Georgia
Vinita Singh, MD
Department ofAnesthesiology
Emory University School ofMedicine
Atlanta, Georgia
Brian Bobzien, MD
Department ofAnesthesiology
Emory University School ofMedicine
Grady Memorial Hospital
Atlanta, Georgia
Trusharth Patel, MD
Department ofAnesthesiology
Emory University School ofMedicine
Atlanta, Georgia
Jerry Kalangara, MD
Department ofAnesthesiology
Emory University School ofMedicine
Veterans Aflairs Medical Center
Atlanta, Georgia

ELSEVIER
ELSEVIER
1600 John F. Kennedy Blvd.
Ste 1800
Philadelphia, PA 19103—2899

PAIN MEDICINE BOARD REVIEW ISBN: 978—0—823—4481 1—6

Copyright © 2018 by Elsevier, Inc. All rights reserved.

No part of this publication may be reproduced or transmitted in any form or by any means, electronic
or mechanical, including photocopying, recording, or any information storage and retrieval system,
without permission in writing from the publisher. Details on how to seek permission, further informa—
tion about the Publisher’s permissions policies and our arrangements with organizations such as the
Copyright Clearance Center and the Copyright Licensing Agency, can be found at our website: www.
elsevier. com/permissions.

This book and the individual contributions contained in it are protected under copyright by the Publisher
(other than as may be noted herein).

Notices

Knowledge and best practice in this field are constantly changing. As new research and experience
broaden our understanding, changes in research methods, professional practices, or medical treat—
ment may become necessary.
Practitioners and researchers must always rely on their own experience and knowledge in evalu—
ating and using any information, methods, compounds, or experiments described herein. In using
such information or methods they should be mindful of their own safety and the safety of others,
including parties for whom they have a professional responsibility.
With respect to any drug or pharmaceutical products identified, readers are advised to check the
most current information provided (i) on procedures featured or (ii) by the manufacturer of each
product to be administered, to verify the recommended dose or formula, the method and duration
of administration, and contraindications. It is the responsibility of practitioners, relying on their own
experience and knowledge of their patients, to make diagnoses, to determine dosages and the best
treatment for each individual patient, and to take all appropriate safety precautions.
To the fullest extent of the law, neither the Publisher nor the authors, contributors, or editors,
assume any liability for any injury and/ or damage to persons or property as a matter of products li-
ability, negligence or otherwise, or from any use or operation of any methods, products, instructions,
or ideas contained in the material herein.

Library of Congress Cataloging-in—Publication Data

Names: Woodbury, Anna, author.


Title: Pain medicine board review / Anna Woodbury, Boris Spektor, Vinita Singh, Brian Bobzien,
Trusharth Patel, Jerry Kalangara.
Description: First edition. | Philadelphia, PA : Elsevier, [2018] | Includes index.
Identifiers: LCCN 2017007004 | ISBN 9780323448116 (pbk. : alk. paper)
Subjects: | MESH: Pain Management—methods | Examination Questions
Classification: LCC RB127 | NLM WL 18.2 | DDC 616/.0472076—dc23 LC record
available at https://1ccn.loc.gov/2017007004

Executive Content Strategist: Michael Houston


Content Development Specialist: Angie Breckon
Publishing Services Manager: Catherine Jackson
Project Manager: Tara Delaney
Design Direction: Brian Salisbury
Working together
LSEVIER
ZZI
Book Aid
International
to grow libraries in
developing countries
Printed in United States of America

Last digit is the print number: 9 8 7 6 5 4 3 2 1


www.clsevier.com o WWW.bookaid.org
Contributors
Jose Avila-Calles, MD, PhD Daniel P. Loren, MD
Assistant Professor, Department of Anesthesiology (Pain Interventional Pain Management Specialist and
Medicine),Jackson Stephens Spine Center, University Anesthesiologist, Orlando, Florida
of Arkansas for Medical Sciences
Joshua Meyer, MD
Brian Bobzien, MD Interventional Pain Medicine, Dothan, Alabama
Assistant Professor, Department of Anesthesiology (Pain
Medicine), Emory University School of Medicine, Preeti Narayan, MBBS
Director of Pain Services, Grady Memorial Hospital, Regional Anesthesia and Acute Pain Fellow, Department
Atlanta, Georgia of Anesthesiology, Emory University, Atlanta, Georgia

Eashwar Balu Chandrasekaran, MD, MSc Trusharth Patel, MD


Clinical Assistant Professor, Department of Emergency Assistant Professor, Department of Anesthesiology (Pain
Medicine, IUH Methodist Palliative Care Services, Medicine), Emory University School of Medicine,
Indiana University School of Medicine, Indianapolis, Atlanta, Georgia
Indiana
Joshua Scott Powers, MD
Shivang Vinod Desai, MD Bain Complete Wellness, Tampa, Florida
Associate, Departments of Interventional Pain and
Anesthesiology, Geisinger Clinic Moises Adrian Sidransky, MD
Danville, Pennsylvania Interventional Pain Management, East Texas Medical
Center, Tyler, Texas
Lisa Guo Foster, MD
Associate Professor, Department of Orthopedics, Emory Vinita Singh, MD
Orthopaedics 8c Spine Center, Atlanta, Georgia Assistant Professor, Director of Cancer Pain Services,
Department of Anesthesiology (Pain Medicine), Emory
Lynn Marie Fraser, MD University School of Medicine, Atlanta, Georgia
Pain Medicine Fellow, Department of Anesthesiology,
University of North Carolina School of Medicine, Boris Spektor, MD
Chapel Hill, North Carolina Program Director, Emory Pain Fellowship, Assistant
Professor, Department of Anesthesiology (Pain Medicine),
Jerry Kalangara, MD Emory University School of Medicine, Atlanta, Georgia
Assistant Professor, Department of Anesthesiology (Pain
Medicine), Emory University School of Medicine, Liliana Viera—Ortiz, MD
Veterans Affairs Medical Center, Atlanta, Georgia Palliative Medicine, Trauma Hospital, University of
Puerto Rico, Medical Center, San Juan, Puerto Rico
Brian H. Keogh, Jr., MD
Banner Health Interventional Pain and Spine Anna Woodbury, MD, C.Ac
Management Clinic, Greeley, Colorado Assistant Program Director, Emory Pain Fellowship,
Assistant Professor, Department of Anesthesiology (Pain
Zachary Leuschner, MD Medicine), Emory University School of Medicine,
Pain Management Physician, Baylor Scott 8c White Health, Division Chief, Veteran’s Affairs Medical Center, Atlanta,
Marble Falls, Texas Georgia
Preface

The field of pain medicine is an ever-developing and expand- This question book is not intended as a stand-alone re-
ing field with an inherently multidisciplinary nature. The source, but as an aid to guide studying and highlight key
field continues to advance with research into newer, safer, points in pain management. It is a companion book to Prac—
and more comprehensive techniques for the management of tical Management of Pain, edited by Honorio Benzon et al.
pain. In the face of a widespread opioid epidemic, physicians Practical Management of Pain stands out as a comprehensive
from a variety of fields have become intensely interested in resource for those interested in the study of pain; it is ap—
nonnarcotic management of acute and chronic pain condi- propriate both for those seeking board certification as well
tions and learning how to appropriately assess opioid risk. As as for those who simply wish to gain a deeper understanding
those who study pain know, there are many varieties of pain of pain and its various treatments. Healthcare practitioners
and many ways to target these individual pain sources and from a variety of fields would benefit from information
interrupt their mechanisms of development. found in these books. In assessing currently available books
This question book highlights some key concepts in the regarding pain medicine for our fellows to use for self-study
pathophysiology and treatment of pain. It was compiled by and board review, we found that most books on the market
physicians from specialties and subspecialties including An— were inadequate and inherently flawed. We therefore en—
esthesiology, Emergency Medicine, Pain Medicine, Pallia- couraged them to read Practical Management of Pain and, as
tive Care, Physical Medicine and Rehabilitation, and Re- a group interested in furthering education and understand-
gional Anesthesia/Acute Pain, and it also includes editors ing regarding pain medicine, came together (fellows and
with expertise and special interests in Cancer Pain, Integra- faculty) to develop a question book based off of this com-
tive Medicine, and Pediatric Pain. Questions were written prehensive text. As such, this question book should be used
primarily by fellows during their subspecialty training at as an adjunct to specifically target areas in need of further
Emory University School of Medicine and reviewed/ edited study, whether for board preparation or simply as a “test
by their attendings. Editors represent four separate institu- your knowledge” guide to accompany Practical Management
tions associated with Emory University School of Medicine, ofPain.
bringing with them a wide range of backgrounds and exper-
tise with diverse patient populations.

vi
Acknowledgments

The editors would like to acknowledge helpful discussions (Emergency Medicine 8: Palliative Care), Lynn O’Neill
and input from their colleagues in multiple departments at (Geriatrics, Internal Medicine & Palliative Care), Michael
Emory University School of Medicine. The multidisciplinary Silver (Neurology), Taylor Harrison (Neurology 8c Electrodi—
pain fellowship at Emory and the development of this book agnostics), Natalie Strickland (Pediatric Pain), Jennifer
could not exist without the willingness and enthusiasm to Steiner (Pain Psychology), Nadine Kaslow (Pain Psychology),
teach that has come from the cohesive groups of individuals Howard Levy (Physical Medicine 8c Rehabilitation), William
within these departments. These departments include Anes- Beckworth (Physical Medicine & Rehabilitation),]ose Garcia
thesiology, Emergency Medicine, Hematology 8c Oncology, (Physical Medicine 8c Rehabilitation), Randy Katz (Occupa—
Interventional Radiology, Neurology, Palliative Care, Physical tional Medicine & Physical Medicine 8c Rehabilitation), Scott
Medicine and Rehabilitation, Primary Care, Psychiatry, Psy- Firestone (Psychiatry), Walter Carpenter (Radiology). And of
chology, Radiology, and many others. Specific individuals de— course, the field would not advance without the aid of those
serving of thanks for their support and their commitment to who have dedicated themselves to advancing research in
medical education include Laureen Hill (Chair, Department pain and neural networks, including Paul Garcia, Wei Huang,
of Anesthesiology), Anne Marie McKenzie—Brown (Director, Vitaly Napadow, Bruce Crosson, Ling Wei, and Shan Ping Yu.
Center for Pain), Colette Curtis (Acute Pain), Tammie Quest
(Emergency Medicine & Palliative Care), Paul Desandre Anna Woodbury, MD

vii
GENERAL CONSIDERATIONS

The History of Pain Medicine

QUESTIONS
1. Which of the following anesthetics was administered for 3. Which of the following organizations is multidisci-
labor pain to Queen Victoria in 1874 and subsequently plinary, with members including but not limited to
cited as legitimizing analgesia during labor? physicians, dentists, psychologists, nurses, and physical
A. Chloroform therapists?
B. Nitrous oxide A. The American Academy of Pain Medicine (AAPM)
C. Cocaine B. The International Association for the Study of Pain
D. Morphine (IASP)
E. Procaine C. The International Spine Intervention Society
(SIS, formally ISIS)
2. Which of the following theories combines both physical D. The American Society of Interventional Pain
and psychological aspects of pain perception and is Physicians (ASIPP)
credited with revolutionizing pain research? E. The American Society of Regional Anesthesia
A. Pattern Theory (ASRA)
B. Sensory Interaction Theory
C. Gate Control Theory
D. The Fourth Theory of Pain
E. Specificity Theory

ANSWERS

I. A. Queen Victoria was given chloroform byjames Simp- techniques, are based on the Gate Control Theory.
son in 1847 for the delivery of her eighth child, at This theory is cited as ending the debate regarding
which point it became widely accepted that labor pain whether or not the cerebral cortex plays a role in pain.
should be medically managed. Prior to this, it was con— Development of imaging such as PET, fMRI, and
sidered against Christian beliefs to provide or accept SPECT later added credibility to this theory by demon—
analgesia during labor. strating the activation of the cerebral cortex in
response to pain.
2. C. The Gate Control Theory, developed by Melzack
and Wall in 1965, states that nonnociceptive signals 3. B. The International Association for the Study of Pain
can override nociceptive signals, and, as a result, the (IASP) is the largest multidisciplinary international as-
perception of pain is reduced or eliminated. Interven- sociation, with a goal of furthering pain research by in-
tions such as peripheral nerve stimulators, TENS units, tegrating professionals with different backgrounds and
and spinal cord stimulators, as well as biofeedback disciplines.
2 Taxonomy and Classification
of Chronic Pain Syndromes

QUESTIONS
1. The International Association for the Study of Pain D. Serves to exclude new syndromes such as those
(LASP) classification focuses on chronic pain; however, involving painful legs and moving toes
it includes syndromes that are not acute in nature, E. Is entirely psychogenic in nature
including which of the following?
A. Acute herpes zoster . Migraines fall under which of the following diagnostic
B. Burns with spasm categories?
C. Pancreatitis Pain Disorder, Somatoform Persistent

£115.09”?
D. Prolapsed intervertebral disk Pain Disorder, Psychological Origin
E. All of the above Pain Disorder, Malingering
Pain Disorder, Neuropathic
. The classification of chronic pain specifies five axes for Psychological or Behavioral Factor Associated with
describing pain. The second axis is the system most re- Disorders or Disease Classified Elsewhere
lated to the cause of the pain. Which of the following
are systems identified? 7. While defining pain, it is important to recognize that
A. The central, peripheral, and autonomic nervous pain is always a subjective state related to which of the
systems and special senses following?
B. Psychological and social function of the nervous Emotional state
PLUGS”?

system Physical state


C . Respiratory and vascular systems Psychiatric state
D. The musculoskeletal system and connective tissue Psychological state
E. All of the above Social state

3. Which of the following is NOT part of the diagnosis of 8. According to the IASP Taxonomy Committee, chronic
complex somatic symptom disorder? pain is defined as pain that has been present for what
A. Emotional disturbances length of time?
B. Health anxiety 3 months
HUGE”?

C. Excessive amount of time devoted to health concerns 12 months


D. Symptoms and concerns must have lasted 12 months 6 months
E. None of the above 24 months
None of the above
. Based on the ICD—lO classification, a predominant
complaint that is persistent, severe, and distressing that . Consensus generally exists on the meaning or
cannot be explained fully by a physiologic process or definition of which of the following terms?
physical disorder is categorized as: A. Condition
A. Pain Disorder, Somatoform Persistent B. Disease
B. Pain Disorder, Psychological Origin C. Disorder
C. Pain Disorder, Malingering D. Symptom
D. Pain Disorder, Neuropathic E. Syndrome
E. Psychological or Behavioral Factor Associated with
Disorders or Disease Classified Elsewhere 10. All of the following are listed by the IASP as relatively
generalized pain syndromes except:
. The definition of complex regional pain syndrome A. Fibromyalgia
(type 1): B. Phantom Pain
A. Is related to the sympathetic nervous system C. Complex Regional Pain Syndrome
B. Is defined by its clinical phenomena D. Pain of Psychological Origin
C. Is defined solely for research purposes E. Radicular Pain
CHAPTER 2 — Taxonomy and Classification of Chronic Pain Syndromes 3

ANSWERS

l. E. The LASP (International Association for the Study of based on a theoretical relationship to the sympathetic
Pain) focuses on the classification of chronic pain syn- nervous system.
drome, but includes some acute syndromes for compar-
ison (and because these acute conditions can often be- E. Pain that is due to known or inferred psychophysio-
come chronic). Acute herpes zoster, burns with spasm, logic mechanisms, such as muscle tension pain or mi—
pancreatitis, and prolapsed intervertebral disk are all graines, but is believed to have a psychogenic cause
examples of acute pain syndromes that are included. falls under the ICD-lO classification of Psychological or
Behavioral Factor Associated with Disorders or Disease
. E. The classifications are divided into five axes: (l) ana- Classified Elsewhere.
tomic, (2) system, (3) temporal characteristics and pat-
tern, (4) intensity, (5) etiology. The second axis systems D. The definition of pain by the IASP is “an unpleasant
include (a) central, peripheral, and autonomic nervous sensory and emotional experience associated with ac-
and special senses; (b) psychological and social function; tual or potential tissue damage or described in terms of
(c) respiratory and vascular; (d) musculoskeletal and such damage.” This addresses the situation of patients
connective tissue; (e) cutaneous and subcutaneous tissue who appear to have pain but do not have obvious tissue
and glands, gastrointestinal, genitourinary, and other damage and acknowledges that pain is always subjective
organs/viscera; and (g) unknown systems. and psychological, regardless of tissue damage.

. D. To be diagnosed with complex somatic symptom . C. Chronic pain is pain that persists beyond the normal
disorder by DSM-IV criteria, patients must report at healing process. Although the timeframe for this may
least one distressing somatic symptom as well as at least differ in practice and many types of pain become
one of “emotional/ cognitive/behavioral disturbances: chronic or persistent at 3 months, the 6—month division
high levels of health anxiety, disproportionate and was chosen for scientific purposes by the IASP as a
persistent concerns about the medical seriousness of good entry to the patient population treated by pain
the ‘symptoms’ and an excessive amount of time and physicians.
energy devoted to the symptoms and health concerns,”
for at least 6 months’ duration. . D. The words “disorder,” “syndrome,” and “disease”
are all in dispute regarding whether they reflect the
4. A. Persistent Somatoform Pain Disorder is persistent, true phenomena that physicians treat. However, the
severe, distressing pain that cannot be explained fully word “symptom” is not in dispute.
by physiologic mechanisms. Pain during schizophrenia
or depression is not included. 10. E. Relatively generalized syndromes include diffuse or
widespread pain that is poorly localized, such as rheu-
. B. The name of CRPS was changed from RSD based on matoid arthritis, fibromyalgia, polymyalgia rheumatica,
the advice of a special subcommittee. Steps taken have pain of psychological origin, syringomyelia, central
(1) defined CRPS type 1 by its clinical phenomena and pain, CRPS, phantom pain, stump pain, and periph-
(2) developed identifying diagnostic criteria for clinical eral neuropathy. Localized syndromes are divided by
agreement as well as for more stringent research pur- the area affected (head, neck, limbs, thorax, abdomen,
poses. The classification has also helped in understand- spinal/ radicular) .
ing relatively new syndromes. The old name, RSD, was
Organizing an Inpatient Acute
Pain Service

QUESTIONS
1. Which of the following factors is likely to influence C. 50%—60%
postoperative opioid requirements? D. 60%—70%
A. Preoperative pain sensitivity E. 80%—90%
B. Presurgical opioid tolerance or a history of drug
abuse . All of the following are examples of multimodal
C. Psychological factors, including catastrophizing and analgesia EXCEPT:
anxiety A. Neuraxial block and music therapy
D. Age B. IV morphine and fentanyl patch
E. All of the above C. PCA morphine and thoracic epidural
D. Acupuncture and TENS
2. Which is the best intervention for inhibition of surgical E. Femoral nerve block and stress reduction
stress responses?
A. Neuraxial steroids . Which is an important first step in organizing an inpa-
B. Neuraxial local anesthetics tient acute pain service?
C. Perineural local anesthetics A. Enlisting the support of hospital administration and
D. Systemic steroids defining resources
E. None of the above B. Assessment of need
C. Definition of service
. Postoperative pain is identified as one of the major D. Financing and business plan
fears of patients undergoing surgery. What percentage E. Nursing education
of patients consider it to be their primary fear?
A. 30%—40%
B. 40%—50%

ANSWERS

l. E. Achieving satisfactory acute pain management can be 4. B. A time-, energy—, and cost-effective acute pain pro-
challenging. It is often difficult to estimate a patient’s gram should optimally provide multimodal and multidis-
postoperative analgesic requirements. The following ciplinary interventions, including systemic and regional
factors may influence postoperative opioid require— pharmacologic treatments, stress reduction, transcuta—
ments: preoperative pain sensitivity, coexisting medical neous electrical nerve stimulation, music therapy, and
conditions and associated multiple drug administration, acupuncture. Extracting and integrating the relevant
presurgical opioid tolerance or a history of drug abuse, expertise from multiple health care disciplines often
psychological factors (including catastrophizing and allows individualized and optimized pain management.
anxiety), age, and type of surgery. Disciplines commonly involved include psychology,
pharmacy, physical therapy, and nutrition.
. B. Surgical stress responses are best inhibited by neur-
axial administration of local anesthetics; the adminis- 5. A. Enlisting the support of hospital administration and
tration of other agents—systemically, neuraxially, or defining resources are a Vital first step in organizing an
perineurally—appears to contribute little additional inpatient acute pain service. Once the challenge of or—
reduction of the endocrine (metabolic and catabolic) ganizing an acute pain service is accepted, assessment
stress response following operative procedures. of need is mandatory. Once the mission statement has
been formulated in response to the perceived institu-
. C. Inadequacy of pain relief has been highlighted as a tional and community needs, it is necessary to define
quality-of—care measure and a focus of patients’ con- the resources that will be required. The next step in the
cern. In a questionnaire survey, 57% of patients identi- process of organizing an inpatient acute pain service is
fied pain after surgery as their primary fear. to construct the business plan.
Measurement-Based
Stepped Care Approach to
Interdisciplinary Chronic
Pain Management

QUESTIONS
1. Which of the following is FALSE regarding the World 5. All of the following are aberrant drug behaviors
Health Organization cancer pain analgesic ladder? EXCEPT:
A. It is focused on the relief of intensity of cancer pain. A. Self-induced oversedation
B. It incorporates relief of suffering of the cancer pain B. Continuing medication despite report of feeling
patient. intoxicated
C. It emphasizes treating the intensity of pain even at C. Early refill requests
the expense of function. D. Calling the office to report worsening pain
D. It includes three steps in its analgesic strategy. E. Self-directed dose increase
E. The goal of the ladder is complete freedom from
pain. . The 2006 Trends and Risks of Opioid Use for Pain
(TROUP) study found opioid use to be higher in pa-
. Which the following pain treatment domains should tients with mental health disorders and what other
ideally be included in a measurements-based stepped health problem?
care pain treatment algorithm? A. Chronic pelvic pain
A. Physical and emotional function B. Substance use disorders
B. Quality of sleep C. Chronic back pain
C. Risk for chemical dependency D. Postsurgical patients
D. Self-reported quality of life E. Patients with whiplash history
E. All of the above
. All of the following are validated opioid risk scales
. The Patient Health Questionnaire 4 (PHQ—4) is a EXCEPT:
screening tool for depression and anxiety. Which of the A. ORT
following is NOT assessed on this questionnaire? B. COMM
A. Feeling nervous, anxious, or on edge C. SOAPP—R
B. Not being able to stop or control worrying D. DIRE
C. Feeling down, depressed, or hopeless E. DOLOPLUS
D. Having little interest or pleasure in doing things
E. Feeling better off dead 8. Which of the following is true about patient access to
pain specialists?
4. Which of the following is true about daily morphine A. There is an overabundance of pain care providers in
equivalent dose (MED) and relative risk of mortality in the United States today.
patients on chronic opioid therapy? B. There is currently a significant shortage in pain
A. As daily morphine equivalent dose increases, providers relative to the number of people with
mortality risk also increases in tandem. chronic pain.
B. As daily morphine equivalent dose increases, C. The number of patients with chronic pain is cur-
mortality risk tends to plateau. rently well matched to the number of board-certified
C. As daily morphine equivalent dose increases, providers in pain care.
mortality risk decreases. D. In the United States, fewer than 1000 physicians
D. The 50—100 mg morphine equivalent dose has the were board-certified in pain care between 2000 and
highest risk for mortality. 2009.
E. Using between 20 and 50 mg morphine equivalents E. The current shortage in pain care expertise leaves
per day does not increase mortality risk relative to more than 100,000 people with chronic pain for
less than 20 mg daily. every pain specialist in the United States.
6 PART 1 — GENERAL CONSIDERATIONS

. Obstructive sleep apnea risk is thought to increase C. Behavioral health specialist


with the dose of opioid used. Which of the following D. Addiction medicine specialist
is NOT an additional risk factor for obstructive sleep E. Sleep medicine specialist
apnea based on STOP-BANG criteria?
. Hypertension 13. The concept of “adverse selection” in relation to
mvow>

. Snoring substance use disorders and chronic pain opioid


. BMI greater than 35 prescribing argues which of the following:
. Age less than 50 A. The patients at highest opioid risk are being
. Male gender prescribed the highest opioid doses.
B. The patients with highest socioeconomic status are
10. According to the model of measurement—based being prescribed the highest opioid doses.
stepped care, referral to a behavioral health specialist C. The patients with lowest opioid needs are being
is indicated when all of the following are present prescribed the lowest opioid doses.
EXCEPT: D. The patients at highest opioid risk are being
PHQ—Q 2 15 prescribed the lowest opioid doses.
FCC???

PHQ—4 2 5 E. The patients at lowest opioid risk are being


Suicidal ideation prescribed the highest opioid doses.
PTSD
Anxiety 14. What is the morphine equivalent dose (MED) that
prompts a referral to a pain specialist according to
11. A 28-year-old male with chronic pain presents to your the stepped care model?
clinic. Opioid risk screening reveals a personal and A. 0—19 mg MED
family history of cocaine use as well as a psychological B. 20—49 mg MED
history of anxiety. According to the stepped care C. 50—79 mg MED
model, he should be referred to see: D. 80—119 mg MED
A. Pain physician E. Greater than 120 mg MED
B. Physiatrist
C. Behavioral health specialist 15. Physical medicine and rehabilitation referral is
D. Addiction medicine specialist prompted by all of the following EXCEPT:
E. Sleep medicine specialist A. Disability greater than 4 weeks
B. On-the—job pain interference
12. A primary care physician is treating a 26-year-old C. Ineffective return—to—work plan
female with chronic diffuse pain of unclear etiology, D. Roland Morris > 12/ 24
unresponsive to 3 months of conservative management E. Obesity
including opioid escalation. According to the stepped
care model, she should be referred to see:
A. Pain physician
B. Physiatrist

ANSWERS

1. B. The WHO cancer pain analgesic ladder is focused 6. B. Presence of substance use disorder in addition to
strictly on alleviating the intensity of pain and does not mental health disorders predisposed patients to higher
incorporate suffering of the cancer pain patient in its opioid use in the TROUP study.
analgesic strategy. Other answer choices listed are true
statements regarding the ladder. 7. E. The first four options are validated opioid risk
screening tools: Opioid Risk Tool (ORT), the Screener
2. E. All of the listed pain treatment domains should and Opioid Assessment for Patients with Pain—Revised
ideally be included in a stepped care pain treatment (SOAPP—R) ; the Current Opioid Misuse Measure
algorithm. (COMM); and the Diagnosis, Intractability, Risk, and
Efficacy (DIRE). The DOLOPLUS scale, though vali-
3. E. The question regarding suicidality comes from the dated, is used for behavioral pain assessment in elderly
PHQ—Q screening tool rather than the simpler PHQ—4 with verbal communication problems, not for opioid
questionnaire. risk screening.

4. A. Relative risk of mortality with chronic opioid therapy . B. There is currently a significant shortage of
increases in parallel with escalating morphine equiva- board-certified pain physicians in the United States
lent dose. relative to the number of patients with chronic pain.

5. D. Contacting the office to report worsening pain is . D. Age greater than 50 is a risk factor for OSA per the
considered appropriate behavior meriting reevaluation. STOP-BANG criteria.
CHAPTER 4 — Measurement-Based Stepped Care Approach to Interdisciplinary Chronic Pain Management

10. E. While anxiety is often associated with chronic pain, 13. A. The patients at highest opioid risk are being
its presence alone is not considered sufficient for psy- prescribed the highest opioid doses.
chiatrist referral according to the stepped care model
unless deemed poorly controlled despite conservative 14. E. Greater than 120 mg MED.
measures.
15. E. All are true except obesity, which is one of the risk
11. D. This patient has an opioid risk tool score 28 and factors for sleep medicine referral.
thus merits referral to an addiction medicine specialist.

12. A. This patient merits referral to a pain medicine


specialist for further evaluation.
5 The Health Care Policy
of Pain Management

QUESTIONS
1. Which of these statements is part of the International basic safety profile and pharmacokinetics of the drug
Association for the Study of Pain (IASP) Declaration of in human subjects?
Montreal? A. Preclinical Investigational New Drug (IND) application
A. Only pain that has been objectively verified must be B. Phase I
treated. C. Phase II
B. Pain must be treated by a medical doctor specialty- D. Phase III
trained in pain management. E. Phase IV
C. Patients should receive analgesia based on their
ability to pay. 3. Which of the following is NOT identified as a barrier to
D. Access to effective pain management is a basic access to pain treatment around the world?
human right. A. Education of patient population
E. Patients must have a history negative for substance B. High cost of treatment
abuse to qualify for opioids. C. Lack of government policy on pain treatment and
drug supply
2. Which of the following phases of the Food and Drug D. Poor training of health care workers
Administration Center for Drug Evaluation and Re— E. Fear among health care workers of legal action
search (FDA CDER) approval process focuses on the

ANSWERS

I. D. The Declaration of Montreal, published in 2010, states 0 Phase III: Confirmation of the safety and effective-
that effective pain management is a basic human right. ness of the drug, its dosages, and drug interactions
It underscores the importance of government to enact in 1000—4000 human subjects.
legislature that promotes access to pain management 0 Phase IV: New or expanded use for patient population
for all. It consists of three articles, including: and long—term risks vs. benefits.
0 Article I: The right of all people to have access to
pain management without discrimination. . A. Access to appropriate pain management worldwide
0 Article 2: The right of people in pain to have their remains largely inadequate. Seventy-eight percent of
pain acknowledged and to be informed about how morphine consumed in 2010 went to only six countries.
it can be assessed and managed. Inexpensive oral medications are unobtainable in many
0 Article 3: The right of all people with pain to have countries. Reasons cited include:
access to appropriate assessment and treatment 0 Failure of governments to put functioning drug supply
of the pain by adequately trained health care systems in place.
professionals. 0 Failure to enact policies on pain treatment and
palliative care.
2. B. The phases of the FDA CDER approval process are 0 Poor training of health care workers.
tiered as follows: 0 Existence of unnecessarily restrictive drug control
0 Preclinical IND application: Animal data used to regulations and practices.
justify testing of the drug in humans. 0 Fear among health care workers of legal sanctions
0 Phase I: Basic safety profile and pharmacokinetics of for legitimate medical practice.
the drug in 50—100 human subjects. 0 Unnecessarily high cost of pain treatment.
0 Phase II: Drug dosage, efficacy, and safety in 100—500
human subjects.
Quality Assessment and 6
Improvement and Patient
Safety in the Pain Clinic

QUESTIONS
1. The majority of errors that take place in health care are C. Lack of evidence-based benchmarks or national “best
due to which of the following? practices”
A. Patient factors D. Lack of cooperation from patients
B. System defects E. Lack of government regulation to support such
C. Provider carelessness programs
D. Equipment malfunction
E. Resource misallocation 3. Which of the following tools has been shown to signifi-
cantly lower the death rate, reduce patient complica-
2. Which of the following is identified as the biggest hurdle tions, and is especially helpful in emergencies?
to establishing Continuous Quality Improvement (CQI) A. E-conferencing/ telecommuting
in pain medicine? B. Speech recognition software
A. Lack of additional reimbursement to justify higher C. Electronic prescriptions
quality care D. Safety checklist
B. Lack of support from hospital administration E. Patient portals

ANSWERS
1. B. According to the Institute of Medicine (IOM), a ma- 3. D. Using a checklist has been shown to reduce patient
jority of errors in health care are more likely attributable complications, which is thought to be due to decreases
to system defects (as opposed to individual errors). in human error. Data shows that simply implementing a
safety checklist can substantially reduce patient mortality
2. C. Because evidence available in peer-reviewed litera- rates. Checklists are critical in emergency situations,
ture is insufficient for chronic pain interventions, estab- such as local anesthetic toxicity, where a checklist/
lishing a CQI in the field of pain medicine largely relies protocol should be readily available.
on consensus expert opinion instead.
Education, Training, and
Certification in Pain Medicine

QUESTIONS
1. Which of the following describes the design of current D. Introduction of devices to the market often outpaces
pain medicine training programs? practitioner familiarity.
Comprehensive and multidisciplinary E. Creation of new devices becomes cost prohibitive for
cow?

. Emphasis on pharmacologic-only treatment manufacturers.


Centered around principles from anesthesiology
. Incorporating multidisciplinary principles from three 3. Which of the following is a problem in using interven-
specialties including anesthesiology, psychiatry, and tional techniques in pain medicine?
neurology A. The procedures are only suitable for a small subset
E. Focus on objective physical exam findings of patients.
B. Randomized, controlled clinical trials are still lacking
2. Which of the following is a consequence of the Food for many interventional treatments.
and Drug Administration’s (FDA) 510(k) “substantially C. There is a lack of patient cooperation with adhering
similar device” process? to the schedule of frequency required.
A. There must be data from >1000 patients demonstrat- D. The complications from most of the procedures are
ing device efficacy. such that the risk outweighs the benefit.
B. There must be data from >1000 patients demon— E. Patients are unwilling to try novel procedures.
strating device safety.
C. Guideline stringency hinders device innovation.

ANSWERS

1. A. The original fellowship training programs in pain market if the FDA deems the risks and benefits are
medicine were extensions of the department of anesthe- comparable to a device that has been previously ap-
siology. Since that time, there has been a broadening proved. The FDA’s 510(k) “substantially similar device”
of specialties seeking training in pain medicine. In an process can require little additional efficacy data. In
effort to standardize and ensure quality of pain medi- some cases, practitioners are overwhelmed with new
cine training programs, four specialties have agreed to technology available and are not able to keep pace with
ACGME requirements for fellowship programs, includ- the skills and knowledge required to use the device.
ing anesthesiology, neurology, physical medicine and
rehabilitation, and psychiatry, since 2007. This collabo- 3. B. The trend in medicine favoring evidence—based therapy
ration between specialties echoes the views of the has highlighted a relative lack of randomized controlled
American Academy of Pain Medicine and the American trials in the pain medicine subspecialty to validate treat-
Board of Pain Medicine, emphasizing a multidisciplinary ments. At the same time, patients with chronic pain re-
and multimodal approach to pain medicine. quire treatment and are often willing to try novel proce—
dures. Practitioners must monitor their own outcomes to
. D. In order to expedite innovation and the introduc- aid in making decisions in patient care as well as use best
tion of new technology, devices can be cleared for judgment of risk vs. benefit for each individual patient.

10
BASIC CONSIDERATIONS

Pain Pathways: Peripheral,


Spinal, Ascending, and
Descending Pathways

QUESTIONS
1. What is the primary termination site for sensory integra- 5. Prolonged membrane hyperpolarization has what effect
tion in the pain pathway? on the role of GABA—B receptors?
A. Thalamus A. Change from inhibition to more inhibition
B. Medulla B. Change from inhibition to excitation
C. Basal ganglia C. Change from excitation to inhibition
D. Dorsal columns D. Increased threshold for depolarization
E. Frontal cortex E. No effect

. What are the small unmyelinated axonal fibers with slow 6. Which of the following is the primary afferent excitatory
conduction velocities that relay noxious input from the neurotransmitter?
skin and other tissues to the central nervous system? A. Substance P
A. A-delta fibers B. CGRP
B. A—beta fibers C. IL—I
C. C fibers D. Glutamate
D. Free nerve endings E. Norepinephrine
E. Dorsal root ganglia
. Noxious cutaneous input is relayed by which of the
The dorsal horn is anatomically organized in laminae. following lamina projection neurons as the crossed
Small unmyelinated fibers terminate in laminae , spinothalamic tract pathway traveling in the lateral
while large myelinated fibers terminate in laminae and ventrolateral white matter en route to the VPL?
A. II—III, IV—V
B. I—II, III—V
C. III—V, I—II
D. II—IV, I—III
E. V, I—II

. The substantia gelatinosa is associated with which of the . The gray matter of the dorsal horn includes the following
following lamina? Rexed laminae:
A. Lamina I A. I—IV
B. Lamina II B. I—VI
C. Lamina III C. II—IV
D. Lamina IV D. II—VI
E. Lamina V E. I—X

ll
12 PART 2 — BASIC CONSIDERATIONS

9. All of the following are considered ascending medial 10. Which of the following structures most likely contrib-
pain pathways EXCEPT: ute to memory and learning related to painful stimuli
A. Spinoamygdalar as indicated by fMRI imaging studies?
B. Spinohypothalamic A. The VPL thalamus, SI, and S11 cortices
C. Spinoreticular B. The ACC, cerebellum, and lentiform nucleus
D. Spinothalamic C. The insula, cerebellum, and frontal cortex
E. Medial spinothalamic D. The ACC, PCC, SI, and S11 cortices
E. The insula, lentiform nucleus, and periaqueductal
gray

ANSWERS

I. A. The primary termination site for sensory integration D. Glutamate is an excitatory amino acid and plays
is the thalamus. Peripheral nociceptors transmit nox— key roles in neural activation throughout the nervous
ious information to second-order neurons at the spinal system. It is the primary neurotransmitter in afferent
cord and brainstem levels, which are then sent by pro- nociception.
jection neurons of the pain system to integration sites
in the brainstem. Though the primary site for integrat- C. Noxious cutaneous input from lamina I, IV, and V
ing sensory information is the thalamus, many other is relayed by projection neurons along the crossed STT
brain structures are also involved. (spinothalamic tract) in the lateral and ventrolateral
white matter to the VPL (ventral posterolateral) and
. C. C fibers (group IV) are small unmyelinated nocicep- posterior thalamus.
tors with conduction velocities less than 2.5 m/ sec.
A—delta fibers (group III) are small fibers with a conduc- . B. Laminae I—X are all considered gray matter, but the
tion velocity of 4—30 m/ sec and conduct faster because dorsal horn includes only laminae I—VI. Some deeper
of a thin myelin sheath produced by Schwann cells. laminae are also involved in nociceptive processing.
Both of these axonal fibers carry noxious input from
tissue to the CNS. . D. The spinoamygdalar, spinohypothalamic, medial spi-
nothalamic and spinoreticular pathways, and connec-
. B. Small unmyelinated C—fiber nociceptor endings for tions to the anterior cingulate and prefrontal and insu-
somatic sensation are distributed mainly in focused lar limbic cortices are all medial pain pathways. The
areas of laminae I and II, though visceral afferents can spinothalamic tract is a nonoverlapping but parallel
extend multiple segments and are widely dispersed in lateral pathway.
ipsilateral laminae I, II, V, and X, or contralateral V and
X. Large myelinated A—beta fibers carrying nonnocicep- 10. C. Centers for higher processing can modulate aware—
tive input terminate in laminae III—V. ness of and responses to pain as well as regulate emo-
tional, autonomic, and motor responses. The insula,
B. Interneurons of lamina II (substantia gelatinosa) syn- cerebellum, and frontal cortex contribute to avoidance
thesize inhibitory (GABA) and excitatory (glutamate) behaviors and other types of memory and learning re-
neurotransmitters. Opioid receptors are also found on lated to the painful stimulus. The VPL thalamus and
these nerve cells. SI and 811 cortices are somatosensory—proeessing regions.
The ACC (anterior cingulate cortex) may be involved
. B. GABA (y-aminobutyric acid) primarily reduces neu- in interpreting emotional significance of pain via the
ronal excitability and provides presynaptic inhibition, limbic system.
though with prolonged hyperpolarization/nociceptive
input, GABA—B receptors can change their role from
inhibition to excitation, leading to a positive feedback
loop that can establish chronic pain.
A Review of Pain-Processing
Pharmacology

QUESTIONS
1. What are the two key inhibitory neurotransmitters? Which of the following receptors are activated by cold
A. NMDA, glycine or menthol?
B. Somatostatin, substance P A. TRPM8
C. AMPA, glutamate B. TRPVl
D. GABA, glycine C. ASIC
E. Somatostatin, NMDA D. TRPV2
E. TRPVS
. Following a peripheral nerve injury, ongoing pain from
ectopic activity is due to the up—regulation of what type . Which of the following protein kinases phosphory—
of channels? lates the NMDA receptor to lower its threshold for
A. Calcium activation?
B. Sodium A. PKA
C. Potassium B. PKC
D. Chloride C. MAPK
E. Fluoride D. AMPA
E. All of the above
. All of the following are effects of substance P binding
EXCEPT: . Blockage of the facilitated state called “wind-up” has
A. Mast cell degranulation been reported with the use of:
B. Swelling of skin A. Opioids
C. Local erythema B. Prostaglandins
D. Vomiting C. Nitric oxide
E. Vasoconstriction D. NMDA antagonists
E. 5-HT3 inhibitors
. After nerve injury there is an increase in axonal excit—
ability that is associated with which of the following? . Chemical injury to a nerve is MOST LIKELY to be
A. Up—regulation of potassium channels and down- characterized by:
regulation of sodium channels A. Reddening at the site of the stimulus
B. Up—regulation of sodium channels and down-regulation B. An initial burst of afferent firing
of potassium channels C. Increased capillary permeability
C. Down-regulation of sodium and potassium channels D. Local arterial dilation
D. Up—regulation of sodium and potassium channels E. Regional hyperalgesia
E. Up—regulation of calcium channels and down-
regulation of sodium channels 10. Which of the following occurs after nerve injury?
A. Increased local catecholamine release
. All of the following mediators are released with tissue B. Alpha-adrenergic antagonists increase excitation of
injury and depolarize and sensitize primary afferent the injured axon
terminals EXCEPT: C. Down-regulation of alpha-l-adrenergic receptor
A. TNF—alpha expression
B. Substance P D. Decreased local catecholamine release
C. Bradykinin E. Decreased activity at the DRG or injured neuroma
D. Histamine
E. Glycine

l3
IA PART 2 — BASIC CONSIDERATIONS

ANSWERS

1. D. GABA and glycine act on GABA—A/GABA—B and gly— 5. E. Following tissue injury, a variety of cytokines and
cine receptors, reducing excitation. Glycine is an inhibi- inflammatory neurotransmitters are released, leading
tory amino acid. Somatostatin is classified as an inhibi- to sensitization. Glycine, however, is an inhibitory neu-
tory hormone with a range of functions throughout the rotransmitter and has not been shown to result in
body including modulating peripheral inflammation, increased afferent sensitization.
the gastrointestinal tract, and the brain. The NMDA
(N—methyl—D—aspartate) receptor requires glutamate or 6. A. Transducer channels on afferent terminals have dif—
aspartate binding for activation. Substance P contributes ferent sensitivities to specific stimuli. Some channels
to enhanced nociception. AMPA (d-amino-S-hydroxy- transducing a physical sensation are also activated by
5-methyl—4—isoxazolepropionate) receptors, like NMDA chemicals and can reproduce the physical sensation
receptors, are activated by glutamate. Glutamate is a main when exposed to the chemical. For example, the
excitatory neurotransmitter. GABA (”y-aminobutyric acid) TRPVl (>43OC) vanilloid receptor is activated by
primarily reduces neuronal excitability and provides presyn— capsaicin (heat sensation), while the TRPM8 (QB—28°C)
aptic inhibition, though with prolonged hyperpolarization/ receptor is activated by menthol (cold sensation), and
nociceptive input, GABA—B receptors can change their the TRPA (<17 oC) receptor is activated by mustard oil.
role from inhibition to excitation, leading to a positive
feedback loop that can establish chronic pain. B. It has been shown that PKG (protein kinase C)
phosphorylates sites on NMDA and AMPA receptors,
B. Sodium channel expression is increased in neuromas which can lower threshold for activation and increase
and dorsal root ganglia following nerve injury. Lidocaine membrane permeability.
can block ectopic activity and attenuate hyperpathic
states after nerve injury. There are a variety of sodium E. Through the bulbospinal pathway, C fibers make
channels in primary afferent neurons (Navl.6—Navl.9). contact with neurons that project into the brainstem
Navl.8 and Navl.9 are resistant to a sodium channel contacting serotonergic neurons, which then project
blocker (TTX) and found primarily in G fibers. Variants to the spinal dorsal horn contacting lamina V neurons.
such as Navl.8 are important because, for example, Cells of lamina V in the deep dorsal horn are noted
Navl.8 reverses nerve-injury—evoked pain states in animal for their ability to display a state known as “wind-up.”
models. Mutations in Navl.7 in humans can cause Blockage of this pathway using 5-HT?) inhibitors has
extremely painful conditions, while loss-of—function been reported to reduce “wind-up” facilitation.
mutations can lead to prominent insensitivity. Gain of
function mutations (i.e., to the sodium ion channel B. Following nerve injury, afferent axons display an
alpha subunit gene SCN9A) can result in syndromes initial burst of afferent firing followed by electrical
like erythromelalgia, characterized by severe periodic silence for hours—days and finally the appearance of
pain from blocked blood vessels. spontaneous bursting activity for hours-days. This is
correlated to an initial degeneration of the injured
. E. Substance P acts on the neurokinin—l (NK—l) recep- nerve followed by new sprouting. Answer choices A, C,
tor and is involved in inflammation, pain perception D, and E are associated with tissue injury.
(along with glutamate), and the vomiting centers.
Substance P is also a potent vasodilator in conjunction 10. A. Stimulation of these postganglionic axons and
with nitric oxide. increased catecholamine release can excite the injured
axon and DRG, but this activation can be blocked by
B. Following nerve injury, sodium channel expression alpha—adrenergic antagonism.
has been found to be significantly increased, while po-
tassium (K+) currents have been shown to be reduced,
suggesting down-regulation of K+ channels.
Pain and Brain Changes

QUESTIONS

1. The TRPVl receptor differentiates which type(s) of D. Magnetoencephalography (MEG)


pain? E. None of the above
A. Noxious heat
B. Acidity . A 32-year-old female underwent cesarean section 1 year
G. Gapsaicin prior to presenting to your office. She has a well-healed,
D. All of the above low, transverse scar. She now presents to the pain clinic
E. None of the above with severe pain at the far right end of the scar. She de-
scribes the pain as intermittent sharp, burning, shoot-
. The dominant excitatory neurotransmitter in all ing pain, exacerbated by touch. On exam, you detect a
nociceptors is: knot at the right distal portion of the incision, and
A. Substance P upon light touch she jumps off the table and curses at
B. Serotonin you. What is the MOST LIKELY way you would describe
G. Acetylcholine her pain:
D. Glutamate A. Abnormal/Hyperalgesia
E. Norepinephrine B. Abnormal/Allodynia
C. Neuropathic pain
Intracranial recordings show that the earliest pain- D. Inflammatory pain
induced brain activity occurs where? E. Visceral pain
A. SI cortex
B. SII cortex . It’s determined that the above patient has a neuroma.
G. Thalamus What INCORRECTLY describes the pain that she’s
D. Hypothalamus experiencing?
E. Dorsal horn A. Neuromas generate spontaneous ectopic activity that
directly contributes to the perception of spontaneous
. Enhanced sensation and neural transmission at sites pain.
remote from the site of injury is referred to as: B. Neuromas are sensitive to temperature, mechanical,
A. Hyperalgesia and chemical stimuli.
B. Primary sensitization C. C—type nerve fibers are involved in spontaneous
G. Secondary sensitization ectopic activity.
D. Desensitization D. A—type nerve fibers are involved in spontaneous
E. Allodynia ectopic activity.
E. Noradrenergic sensitization results in separation of
. Which of the following is considered a nonnociceptive sympathetic and sensory neuronal activity.
touch afferent receptor that, following central sensitiza-
tion, may carry peripheral pain signals to the CNS? 9. Which of the following best describes the mechanisms
A. A—delta involved in sensitization?
B. A-beta A. There is decreased release of excitatory neurotrans-
G. G fibers mitters such as glutamate and substance P.
D. A—gamma B. Opioid receptors are down-regulated with neuro—
E. Schwann cells pathic pain.
C. Opioid receptors are down-regulated with inflamma-
. What electrical study or imaging tool can be used to tory pain.
verify the presence of ongoing spontaneous pain in a D. Potassium chloride (KGl) transporter down-regulation
patient? increases the effects of GABA release.
A. Electroencephalogram (EEG) E. Release of substance P and other peptides closes
B. Positron emission tomography (PET) the N—methyl—D-aspartate (NMDA) glutamate-gated
G. Functional MRI (fMRI) channel.

15
16 PART 2 — BASIC CONSIDERATIONS

10. The dimension of pain thought to include interaction 13. You are performing a neurologic sensory exam. The
with previous experience and involve the prefrontal patient shows normal sensation to light touch in the
cortex is termed: distributions of the radial and median nerves and in-
A. Sensory-discriminative creased sensitivity to pinprick. What most accurately
B. Cognitive-evaluative describes this phenomenon?
C. Affective-motivational A. Allodynia
D. Behavioral-cortical B. Hypesthesia
E. Emotional—receptive C. Paresthesia
D. Hyperalgesia
11. Which of the following is true of placebo analgesia? E. Secondary sensitization
A. It cannot be blocked by naloxone.
B. The endogenous opioid system is involved. 14. Chronic pain conditions are consistently associated
C. The somatosensory cortex is involved. with decreased activity in which brain structure(s)?
D. There is no correspondence between placebo A. Prefrontal cortex
analgesia and reward. B. Thalamus
E. All of the above C. Reticular activating system
D. Pons
12. Which of the following is TRUE regarding bone E. All of the above
cancer pain?
A. Substance P and other neuropeptides, such as 15. Based on brain-imaging studies, which of the following
CGRP, are unregulated. is implicated as a cause for the autonomic symptoms
B. Up—regulation of galanin and neuropeptide Y experienced in cluster headaches?
occurs. A. Activation of the first (ophthalmic) division of the
C. Neuropathic pain and bone cancer pain have trigeminal nerve
similar changes with substance P and CGRP. B. Activation of cranial parasympathetic outflow from
D. The greatest change observed in the spinal cord in the seventh cranial nerve
response to metastatic bone cancer pain is activa— C. Cortical spreading depression
tion of astrocytes. D. Activation of the trigeminovascular system
E. All of the above E. Hyperemia in the occipital cortex

ANSWERS

l. D. The TRPVl receptor responds to stimuli including 7. B. While the pain may be described as inflammatory
heat, acidity, and hot pepper capsaicin. These stimuli and/ or neuropathic as well, the patient is clearly expe—
can enhance each other. riencing an abnormal response to mechanical light
touch. This would be described as tactile allodynia, an
2. D. Glutamate is the dominant excitatory nociceptor experience where a typically innocuous sensation such
neurotransmitter. as light touch becomes painful.

3. B. Intracranial recordings show that the earliest pain— 8. E. Neuromas are sensitive to multiple sensations and
induced signals originate near the SH cortex, implicat— generate spontaneous ectopic activity from both A and
ing SH and adjacent insula regions as the primary brain C fibers. Noradrenergic sensitization can result in cou-
areas for receiving nociceptive input. pling of sympathetic and sensory inputs.

4. C. Secondary sensitization leads to enhanced neural 9. B. In sensitization, opioid receptors are down—regulated
transmission distant from the site of injury, thought to with neuropathic pain, though C) up—regulated with
result from reorganized spinal cord nociceptive circuitry. inflammatory pain; A) there is an increased release of
Primary sensitization, in contrast, leads to increased excitatory neurotransmitters; D) KCl receptor down-
transmission near the site of injury. regulation decreases the effects of GABA release; E)
these peptides open the NMDA channel, leading to
5. B. A—delta and C fibers are the primary nociceptors in increased sensitization.
healthy organisms. A—beta afferents are large nonnoci—
ceptive, heavily myelinated touch receptors. However, 10. B. Traditionally, pain perception consists of the following
following central sensitization, input from these touch dimensions: sensory-discriminative, cognitive-evaluative,
receptors can result in the sensation of pain. and affective-motivational. The cognitive-evaluative di—
mension includes previous experiences and cognitive
6. E. Currently, there is no objective measure of brain activ— influences on perception of pain intensity and, tradi-
ity that can conclusively determine whether an individual tionally, the prefrontal cortex is thought to be involved
is in pain. However, fMRI and PET are able to provide in this dimension. Note: Neuroimaging studies have
valuable information regarding the pathologic process— assessed paradigms that do not easily fit into these
ing of nociception and pain perception. three traditional dimensions and tend to discuss
CHAPTER 10 — Pain and Brain Changes 17

cortical areas and brain regions involved in specific sensation. E) Secondary sensitization is enhanced sensa-
functions instead. tion at sites distant from the site of injury.

ll. B. The placebo response involves the endogenous opi- 14. B. Clinical brain imaging studies show reduced activ-
oid system and can be blocked by naloxone. Studies ity in and transmission through the thalamus for
have shown a correspondence between placebo anal- chronic clinical pain (as opposed to experimentally
gesia and reward pathways. The PAG and amygdala are induced acute pain). There is, however, increased ac-
involved, as well as the prefrontal/rostral ACC. tivity in the prefrontal cortex in chronic pain. These
changes support the idea that chronic pain condi—
12. D. A unique set of neurochemical changes occur at the tions are associated with increased involvement of
level of the spinal cord and dorsal root ganglion for brain regions for cognition and emotion. Meanwhile,
each type of pain: inflammatory, neuropathic, or cancer the sensory and nociceptive regions of the brain (thala-
pain. The greatest change in metastatic bone cancer mus) show decreased activity. There is also a reduc-
pain is activation of astrocytes at the spinal cord. tion in neocortical gray matter.

13. D. Hyperalgesia describes the abnormally increased 15. B. Activation of cranial parasympathetic outflow from
response to pain for a stimulus that would typically cause cranial nerve VII is thought to result in the autonomic
pain, but not to the extent observed. A) Allodynia de- symptoms in cluster headaches. A) Activation of V1 is
scribes a painful response to a normally nonnoxious thought to mediate the excruciating unilateral pain
stimulus such as light touch. B) Hypesthesia describes in cluster headaches. Answer choices C—E apply to
diminished sensation. C) Paresthesia describes abnormal migraine headaches.
An Introduction to
Pharmacogenetics In Pain
Management: Knowledge of
How Pharmacogenomics May
Affect Clinical Care

QUESTIONS
1. Polymorphism of an enzyme involved in tetrahydrobiop- C. Codeine
terin (BH4) synthesis in primary sensory neurons of the D. Oxycodone
dorsal root ganglion following nerve injury results in E. Hydrocodone
which of the following?
A. Increased sensitivity to painful stimuli . Analgesics that require breakdown in the liver through
B. Reduced sensitivity to painful stimuli CYP45O to achieve analgesic effects are metabolized by
C. Increased sensitivity to analgesic medications which of the following?
D. Reduced sensitivity to analgesic medications A. CYP1A2
E. Reduced ability to heal following injury B. CYP2D6
C. CYP2C9
2. Which of the following phenotypes is consistent with D. CYP2C19
either multiple copies of a functional allele or an allele E. CYP3A4
with increased gene transcription?
A. Nonmetabolizer . Which of the following is the name for a field of medi-
B. Poor metabolizer cine that includes a patient’s genetic background and
C. Intermediate metabolizer uses the information to predict how a patient will
D. Extensive metabolizer respond in terms of efficacy and side effects when
E. Ultra-rapid metabolizer given a medication?
A. Pharmacognosy
3. A patient with known mutation in the CYP450 enzyme B. Pharmacokinetics
requires analgesic medication. Which of the following C. Pharmacogenomics
are NOT metabolized through this system at standard D. Genesiology
prescribed doses? E. Pharmacodynamics
A. Hydromorphone
B. Tramadol

ANSWERS
1. B. Patients with increased pain tolerance are thought 2. E. Classification of phenotypes for enzymes includes the
to have a polymorphism (a genetic variant) in GTP cy- following:
clohydrolase, which is the rate-limiting enzyme involved 0 Poor metabolizers: Two nonfunctional enzyme alleles.
in tetrahydrobiopterin (BH4) synthesis. BH4 has been 0 Intermediate metabolizers: At least one reduced
shown to be involved in regulation of inflammatory and functional allele.
neuropathic pain. It is estimated that this polymor- 0 Extensive metabolizers: At least one functional allele.
phism is associated with increased pain tolerance and is 0 Ultra-rapid metabolizers: Multiple copies of a func-
present in 15% of the population. Similarly, reduced tional allele or an allele with a promoter mutation
pain tolerance has been linked to polymorphisms in that confers increased transcription of that gene.
other genes. Different responses to analgesics among Phenotype variations can be responsible for the vast
patients can be attributed to genetic variance as well. difference in clinical effect as well as side effects

18
CHAPTER 1 l — An Introduction to Pharmacogenetics in Pain Management: Knowledge of How Pharmacogenomics May AtfectCIinical Care 19

between two patients taking the same weight-based form, for example conversion of codeine to the active
dosage of medication. metabolite morphine.

3. A. Although 40%—50% of medications are metabolized . C. By preemptively identifying patients at risk for ad-
in the liver through the CYP450 family of enzymes, verse side effects or poor efficacy from medication, the
three opioids are not, including hydromorphone, field of pharmacogenomics may improve health care
morphine, and oxymorphone. outcomes and efficiency, including higher success rate
after medication administration, lower incidence of side
. B. Analgesics that are metabolized by CYP450 include effects, and reduction of cost. As a relatively new field, it
codeine, dextromethorphan, oxycodone, and tramadol. shows promise in terms of tailoring medication regi-
Prodrugs require CYP2D6 for conversion to the active mens to each patient.
2 Psychosocial Aspects
of Chronic Pain

QUESTIONS
1. Chronic pain is best understood using which of the C. Positive reinforcement
following models? D. Both A and B
A. Biomedical E. All of the above
B. Biopsychosocial
C. Psychogenic . What is an exaggerated negative orientation toward
D. Secondary-gain actual or anticipated pain experiences?
E. Both A and C A. Self-efficacy
B. Fear avoidance
. In regards to psychological factors that play an important C. Pain catastrophizing
role in the experience of pain, which of the following is D. Anxiety
an affective factor rather than a cognitive factor? E. Negative reinforcement
A. Anger
B. Catastrophic thinking 6. Your patient had significant pain following his physical
C. Beliefs about pain therapy sessions, and now every time he drives by the
D. Self-efficacy facility he becomes tense and his back pain increases in
E. Coping severity. This is considered:
A. Classical conditioning
3. All of the following are examples of passive coping strat- B. Operant conditioning
egies EXCEPT: C. Negative reinforcement
A. Inactivity D. Punishment
B. Distraction E. Fear avoidance
C. Medications
D. Alcohol . According to one study, the target of a patient’s anger
E. Avoidance was most commonly acknowledged to be the:
A. Health care worker
4. According to the operant conditioning principles of re- B. Attorney
inforcement, the following response to a specific behav- C. Insurance company
ior will likely decrease the probability of the behavior D. Patient themselves
recurring: E. Significant other
A. Neglect
B. Negative reinforcement

A NSWE RS

1. B. In chronic pain, there may or may not be an identifi— the response to the illness and treatments, can be
able pathologic process or organic cause. Pain can also understood more completely by using this model.
take a significant emotional toll on the patient leading
to feelings of demoralization, helplessness, hopeless- 2. A. Affective factors are emotions, and in chronic pain,
ness, depression, and anxiety, to name a few. Since peo- they are mainly negative emotions such as depression,
ple rarely live in complete isolation, there is a larger anxiety, and even anger. The other factors listed are
social context that impacts and influences a person’s cognitive factors, which are involved in how someone
chronic pain. Given the multidimensional nature of thinks, perceives, and reasons. Catastrophic thinking is
chronic pain, a biopsychosocial approach is best used defined as an exaggerated negative orientation toward
to understand this condition. This model focuses on the actual or anticipated pain experiences. It can also be
illness as a whole, which is the result of a complex inter- described as a set of maladaptive beliefs. Beliefs about
action of biologic, psychological, and social variables. the meaning of pain influence a person’s expectations
Both the patient’s perception of his/her pain, as well as about pain and can be either positive or negative.

2O
CHAPTER 12 — Psychosocial Aspects of Chronic Pain 2]

Self-efficacy is a personal conviction that one can suc- 5. C. Pain catastrophizing is defined as an exaggerated nega-
cessfully execute a course of action to produce a desired tive orientation toward actual or anticipated pain expe-
outcome. This is a positive factor in chronic pain patients riences. Example: “It’s terrible, and I think it’s never go—
that can affect physical and psychological functioning. ing to get any better.” Fear avoidance, while seemingly
The concept of coping is basically self-regulation of pain similar, is a conditioned response where certain activi-
by using purposeful and intentional acts to help deal, ties, which have caused pain in the past, are avoided
adjust, reduce, or minimize the pain. because of the anticipation that they will cause pain.

. B. Coping is self-regulation of pain by using purposeful 6. A. This is an example of classical conditioning. By


and intentional acts to help deal, adjust, reduce, or experiencing pain following repeated physical therapy
minimize the pain. It can be divided into overt and sessions, the patient became “conditioned” and experi-
covert strategies or passive and active strategies. Passive enced a negative emotional response toward the facility,
coping strategies involve depending on others for help which he associated with the activity. The negative
and restriction of activities, and are related to greater emotional response led to tensing of muscles, which
pain and depression. These include inactivity, avoid- increased his pain, ultimately strengthening the associa-
ance, medications, alcohol, and drugs. Patients utilizing tion. This may eventually lead to fear avoidance, where
only passive strategies tend to absolve themselves of the patient avoids physical therapy (or driving by the
personal responsibility for managing their pain. Active facility) because of the anticipation of pain. Negative
strategies, on the other hand, have been associated with reinforcement and punishment are both principles of
adaptive functioning and improvement in pain. These operant conditioning, which focus on modifying the
include efforts to function despite pain, distracting one- frequency of a given behavior through the use of conse—
self from pain, relaxation techniques, and reassuring quences, either rewarding or aversive.
oneself that the pain will diminish.
. D. There have been studies showing anger is a fairly
4. A. The operant principles of reinforcement include common emotion in patients with chronic pain, and it
positive reinforcement, negative reinforcement, punish- is strongly associated with pain intensity, perceived in-
ment, and neglect. Positive and negative reinforcement terference, and pain behaviors. Okifuji and colleagues
both increase the likelihood of a behavior recurring found that the target of anger was most commonly ac-
(reinforces) but through different consequences— knowledged as oneself (70% of patients). Other targets
rewarding the behavior or preventing/withdrawing included health care providers (60%), significant others
aversive results, respectively. Neglect prevents or with- (39%), insurance companies (30%), employers (26%),
draws positive results, making a behavior less likely to and attorneys (20%). (Okifuji A, Turk DC, Curran SL.
recur. Punishment, although not an option in the Anger in chronic pain: investigations of anger targets
question, also decreases the probability of a behavior and intensity. sychosome Res. 1999;47(1):l-12.)
recurring but by punishing the behavior.
This page intentionally left blank
EVALUATION AND ASSESSMENT

History and Physical


Examination of the Pain Patient

QUESTIONS
1. A 83-year-old man presents for his yearly physical. On D. May be seen with true organic pathology
physical exam, upon stroking the lateral aspect of the E. Definitively differentiates between organic and non-
bottom of his foot you see his great toe extend. This organic pathology
may be a sign of:
A. Lower motor neuron dysfunction . Lumbar discogenic pain:
B. Sensory deficit of the foot A. May be exacerbated by Valsalva
C. Autonomic dysfunction B. Is predominantly axial in nature
D. Upper motor neuron dysfunction C. Is reproduced with straight leg raise testing
E. Normal finding in males D. Can be provoked with Spurling’s maneuver
E. A and B
2. What is the primary benefit of the physician bringing
the patient to the room? . Motor strength testing:
A. Increases patient satisfaction by perceived reduced A. Is purely subjective
wait times B. Tests A—B motor fiber function
B. Allows identification of potential inconsistencies on C. May be verified with a Hoover test for effort
examination D. Is rated 4/ 5 when giveway weakness is noted
C. Reduces the amount of time needed for E. Requires placing muscles at a mechanical
documentation disadvantage
D. Improves staff morale
E. Increases work—flow throughput . Asking a patient to track a moving object in the eight
directions of cardinal gaze test the functions of cranial
3. Hoffman’s sign: nerves:
A. Can be seen in normal individuals A. I, II, III
B. Always represents pathology B. 11, V, VIII
C. Can be affected by SSRI use C. III, IV, VI
D. Is elucidated by squeezing the dorsal aspect of the D. III, V, VIII
third distal phalanx E. IV, V, VI
E. A and C
. A 58—year—old male presents for evaluation of peripheral
4. Greater than 3 out of 5 of Waddell’s signs: neuropathy. He has a history of significant alcohol abuse.
A. Indicate the patient is malingering Chronic alcoholism may also result in the inability to:
B. Necessitate prompt referral to psychological services A. Protrude the tongue
C. Are grounds for dismissal from clinic B. Oppose the thumb

23
24 PART 3 — EVALUATION AND ASSESSMENT

C. Sequentially supinate and pronate the hand C. Rotator cuff tear


D. Shrug the shoulders D. Impingement syndrome
E. Flex the hip E. Labral tear

Which of the following cranial nerves innervates the 14. A 54-year-old male presents with left foot pain. Physical
muscles of mastication? exam reveals Morton’s neuroma. Where is pain in
A. CN IV Morton’s neuroma MOST commonly found?
B. CN V A. First and second toes
C. CN VI B. Second and third toes
D. CN VII C. Third and fourth toes
E. CN VIII D. Fourth and fifth toes
E. Not in the foot
10. A 78-year—old patient presents with neck pain radiat-
ing to the left upper extremity. MRI of the cervical 15. Which nerve is entrapped in tarsal tunnel syndrome?
spine shows C4-5 subarticular disk herniation contact- A. Posterior tibial
ing the exiting C5 nerve root. C5 radiculopathy will B. Dorsalis pedis
result in: C. Deep peroneal
A. Shoulder external rotation weakness, no reflex D. Superficial peroneal
changes, decreased sensation in lateral aspect E. Sural
of arm
B. Forearm flexion and supination weakness, absent 16. When pain is provoked by placing the patient in the
biceps reflex, decreased sensation in lateral aspect prone position with the knee flexed to 90 degrees and
of forearm downward force placed on the heel, this is best described
C. Shoulder abduction, extension and flexion weak- as a positive:
ness, absent biceps reflex, decreased sensation in A. Drawer test
lateral aspect of forearm B. Patellar femoral grinding test
D. Forearm extension weakness, absent triceps reflex, C. Apley’s compression test
decreased sensation in medial aspect of arm D. FABER test
E. Forearm flexion in the mid-prone position, absent E. Lasegue’s test
brachioradialis reflex, decreased sensation in medial
aspect of forearm 17. Normal thoracolumbar spine range of motion
consists of:
11. A 23-year-old professional football player presents to A. 90 degrees of forward flexion/3O degrees of back
the pain clinic for evaluation of knee pain that started extension/ 60 degrees of lateral rotation
after he was hit on the side of the knee during a tackle. B. 90 degrees of forward flexion/ 15 degrees of back
A positive anterior drawer sign indicates damage of extension/45 degrees of lateral rotation
which of the following structures? C. 80 degrees of forward flexion/3O degrees of back
A. Medial meniscus extension/7O degrees of lateral rotation
B. Posterior cruciate ligament D. 90 degrees of forward flexion/ 15 degrees of back
C. Patellar tendon extension/ 60 degrees of lateral rotation
D. Anterior cruciate ligament E. None of the above
E. Medial collateral ligament
18. A 62-year-old male presents with Sl radiculopathy. He
12. A 54-year-old female presents with neck pain radiating is able to perform five toe raises while standing and
to the left shoulder. Which of the following is most lightly holding onto something for balance. You con-
specific for C5—6 root compression? clude that he has Grade _ gastrocnemius strength.
A. Left neck pain with simultaneous right lateral flexion A?
and extension of the neck B. 3
. Pain with abduction of the left arm C. 4
ODS

. Pain relief by resting the forearm on the head on D. 5


the left side E. None of the above
. Pain with abduction of the right arm
NC

. Pain relief by resting the forearm on the head on 19. A patient presents with an inability to evert the foot.
the right side What nerve root is most likely damaged?
A. L3
13. A 17-year-old baseball player presents with left shoulder B. L4
pain that is worsened on abduction and elevation. C. L5
These exam findings are consistent with: D. Sl
A. Subdeltoid bursitis E. S2
B. Bicipital tendonitis
CHAPTER 13 — History and Physical Examination of the Pain Patient 25

20. With respect to spinal examination: 24. The following is FALSE about coordination and
A. Lumbar discogenic pain is associated with intolerance balance testing related to the cerebellum:
of the standing position. A. The vermis controls axial coordination and balance,
B. In cervical arthropathy, positional change will whereas the hemispheres coordinate the limbs.
improve range of motion of the cervical spine. B. Vermis integrity can be tested by asking the patient
C. Pain with back extension and lateral rotation to walk “heel—to-toe” one foot in front of the other.
suggest facet joint arthropathy. C. Hemispheric function can be tested with finger-to-
D. The Spurling maneuver is highly sensitive for nose skills.
confirming a diagnosis of cervical radiculopathy. D. Vermian function can be assessed by heel-to—shin tests.
E. Normal thoracolumbar spine range of motion is E. Gait and balance are functions of vermis integrity.
45 degrees in forward flexion.
25. Which muscle is correctly matched with the appropri-
21. The “corneal blink” reflex is carried out by an afferent ate action, nerve root, and nerve?
and efferent pathway belonging to which of the follow- A. Infraspinatus: shoulder external rotation, C5-6,
ing nerves, respectively? suprascapular
A. Optic and abducens B. Biceps: forearm flexion/supination, C5-6, radial
B. Trochlear and V2 of the trigeminal C. Brachioradialis: wrist extension, C6-7, radial
C. Trigeminal and facial D. Flexor carpi ulnaris: wrist flexion and ulnar devia—
D. Facial and VI of the trigeminal tion, C6-7, ulnar
E. Scleral branch of the optic and blefaro-retractive E. Adductor pollicis longus: thumb abduction, C7—8,
branch of the facial median

22. The “gag” reflex is carried out by an afferent and 26. Which of the following is true regarding the vestibulo—
efferent pathway belonging to which of the following cochlear nerve?
nerves, respectively? A. Nystagmus on eye movement testing may be a sign
A. Vagus and glossopharyngeal of vestibular dysfunction.
B. Chorda timpani (facial) and glossopharyngeal B. Testing of the nerve include the Rinne, Weber, and
C. Superior laryngeal branch of vagus and recurrent Snellen tests.
pharyngeal of vagus C. In patients with sensorineural deafness and normal
D. Glossopharyngeal and vagus hearing, bone conduction is better than air
E. Cranial portion of spinal and vagus conduction.
D. In unilateral sensorineural hearing loss tested
23. The following is TRUE about the Hoover test: with the Weber test, sound is heard better in the
A. It is used to test physiologic weakness of the lower affected ear.
extremities. E. It is cranial nerve IX.
B. A positive test occurs when the patient exerts down-
ward force on the heel of the “nonparetic” leg 27. Match the deep tendon reflex with the correct nerve
when asked to elevate the paretic leg. root level:
C. A positive test “rules out” psychogenic A. Biceps, C5-6
etiology of the patient’s limb weakness. B. Brachioradialis, C7—8
D. The test is useful to differentiate between proximal C. Triceps, C5-6
muscle weaknesses from ascending paresis. D. Quadriceps femoris, Ll-2
E. A positive test occurs when the patient fails to exert E. Gastrocnemius/Achilles, L4—L5
downward force on the heel of the “nonparetic” leg
when asked to elevate the paretic leg.

ANSWERS

l. D. Babinski’s and Hoffman’s signs may be present in serotonin reuptake inhibitor antidepressants, a subtly
patients with upper motor neuron dysfunction. The positive Hoffman’s may be present and considered
Babinski reflex is normal in children 0—2 years old. normal.

. B. When the examining physician brings the patient . D. There are five Waddell’s signs; presence of any three
back to the room, it allows them to assess mobility and of these five signs are suggestive of a nonorganic pain
gait. It also allows observation of function outside of the etiology. However, Waddell’s signs may not be able to
“formal” exam setting, which could potentially present accurately distinguish between organic and nonorganic
different findings. causes of pain, according to recent findings, as Waddell’s
signs can also be present even when there is an organic
. E. Upper motor neuron lesions can cause hyperreflexia, cause to pain. Findings on exam should be considered
which may be evaluated by Babinski’s and Hoffman’s in context of other information obtained from the pa-
testing. In young women and individuals taking selective tient’s history and additional diagnostic studies.
26 PART 3 — EVALUATION AND ASSESSMENT

. E. Pain provoked by forward flexion is typical of disco- fourth toes, and less commonly between the second
genic or vertebral body pain, as flexion causes axial and third toes. The pain is reproducible on palpation
loading. Lumbar discogenic pain is often only in the of the space between the metatarsal heads.
axial spine, associated with inability to sit for long peri-
ods, and increased pain with coughing, sneezing, and 15. A. The posterior tibial nerve in tarsal tunnel syndrome
Valsalva. is trapped beneath the laciniate ligament, leading to
pain and paresthesias in the foot and toes.
C. Muscle strength testing depends on patients’
understanding and effort. The Hoover test can 16. C. Apley’s compression or grinding test is used to eval—
help detect psychogenic weakness of the lower uate for medial and lateral meniscal tears. This is the
extremities. test described in the question stem.

. C. Cranial nerves III (ophthalmic), IV (trochlear), and 17. A. For the thoracolumbar spine, normal range of
VI (abducens) control eye movement and can be motion is 90 degrees of forward flexion, 30 degrees
tested by asking the patient to track a moving object in of back extension, and 60 degrees of lateral rota-
the eight positions of cardinal gaze. tion. There is also normally 25 degrees of lateral
flexion. Normal range of motion of the cervical
C. Rapid alternating movements (e.g., sequential hand spine differs.
pronation and supination) are adversely affected in
patients with cerebellar disease. Chronic alcohol abuse 18. C. Muscle strength is commonly assessed with the
can cause acquired cerebellar degeneration. Medical Research Council scale. Grades 1—3 are
relatively objective and less prone to variation. The
. B. The mandibular division of the trigeminal nerve gastrocnemius muscle presents unique challenges be-
also supplies the muscles of mastication (i.e., tempora— cause, in the case of the gastrocnemius, a Grade 3 =
lis, masseter, medial and lateral pterygoid muscles). able to perform one toe raise with the patient standing
and lightly holding onto something for balance (requires
10. C. The C5-6 nerve roots supply: (1) suprascapular more patient participation). Grade 4 = able to perform
nerve (innervates infraspinatus muscle, which helps five toe raises, and Grade 5 = able to perform 10 toe
with shoulder external rotation), (2) axillary nerve (in- ra1ses.
nervates deltoid muscle, which helps with shoulder ab-
duction, extension, and flexion), and (3) musculocu— 19. C. Foot eversion involves the peroneus longus, sup-
taneous nerve (innervates biceps muscle, which helps plied by the superficial peroneal nerve, which has its
with forearm flexion and supination). The biceps ten- source from the L5 nerve root.
don reflex is innervated by C5—6, brachioradialis ten—
don reflex by C6, and triceps tendon reflex by C7-8. 20. C. Pain from back extension and lateral rotation is
C5 supplies the lateral aspect of the arm, and C6 sup- suggestive of facet arthropathy (results in zygapophy—
plies the lateral aspect of the forearm, thumb, and lat- seal joint loading). The Spurling maneuver is highly
eral half of index finger. specific for confirming the diagnosis of cervical radicu-
lopathy. Lumbar discogenic pain is often restricted to
11. D. Movement of the tibia forward relative to the rest of the axial spine and is associated with intolerance of
the leg constitutes a positive anterior drawer sign and the sitting position and pain provoked by coughing,
suggests a tear of the anterior cruciate ligament. sneezing, and Valsalva maneuver.

12. C. A positive abduction tension release sign is highly 21. C. The trigeminal nerve provides the afferent
specific for distinguishing shoulder pathology from (ascending, sensory) limb and facial nerve provides
C5—6 root compression. This test involves having the efferent (descending, motor) limb of the corneal
patient abduct the affected arm and rest the forearm blink reflex.
on top of the head. With shoulder pathology, this will
cause pain, but with lower cervical root compression, 22. D. Cranial nerve IX, the glossopharyngeal nerve,
this typically reduces the radicular pain. The Spurling provides the afferent limb of the gag reflex (taste in
maneuver can also help to distinguish cervical pathol- posterior third of tongue and sensation in pharynx).
ogy from other types of pain. Cranial nerve X, the vagus nerve, innervates the effer-
ent limb of the gag reflex (pharyngeal and laryngeal
13. D. Pain on shoulder abduction and elevation is seen muscles).
with impingement syndrome due to entrapment of
soft tissue between the humeral head and the cora- 23. E. The Hoover test helps for evaluating psychogenic
coacromial arch. Other conditions such as bursitis and weakness of the lower extremities. A positive test oc—
rotator cuff pathology can be present at the same time. curs when the patient fails to exert downward force on
the heel of the “nonparetic” leg when asked to elevate
14. C. Morton’s neuroma is characterized by pain between the paretic leg.
the metatarsal bones, usually between the third and
CHAPTER 13 — History and Physical Examination of the Pain Patient 27

24. D. Cerebellar function can be divided into midline/


vermal and hemispheric functions. The vermis con-
trols axial coordination and balance (gait and standing
balance); the hemispheres coordinate the limbs
(finger—to-nose and heel—to-shin tests).

25. A.

Upper Extremity Muscles: Innervation and Action


Muscle Action Nerve Root Nerve
Infraspinatus Shoulder external rotation (15-6 Suprascapular
Deltoid Shoulder abduction, extension, and (35-6 Axillary
flexion
Biceps Forearm flexion and supination (15-6 Musculocutaneous
Triceps Forearm extension (37-8 Radial
Brachioradialis Forearm flexion in the mid-prone C6 Radial
position
Extensor carpi radialis longus, Wrist extension (36-7 Radial
and brevis
Flexor carpi ulnaris Wrist flexion with ulnar deviation C8—Tl Ulnar
Flexor digitorum profundus Flexion at the distal interphalangeal C7—8 Anterior interosseus branch of
joints the median nerve
Abductor pollicis brevis Abduction of the thumb C8 Median
Adductor pollicis longus Abduction of the thumb C8-T1 Ulnar

26. A. Nystagmus noted on eye movement testing may be


a sign of vestibular dysfunction (cranial nerve VIII, ves-
tibulocochlear nerve, mediates hearing and balance).

27. A.
Nerve Root Innervation of the Deep Tendon Reflexes
Muscle Tendon Nerve Boat Level
Biceps (15—6
Brachioradialis C6
Triceps (17-8
Quadriceps femoris L34
Gastrocnemius/Achilles tendon Sl-2
II 4 Electromyography and Evoked
Potential s

QUESTIONS
1. What comprises a motor unit? C. 5 mm
A. A group of muscle fibers innervated by a single ante- D. 15 mm
rior horn cell E. Cannot calculate with the given information
A single muscle fiber innervated by an axonal branch
COP“

A group of muscle fibers that make up a single muscle Which of the following conditions is most likely to
A group of muscle fibers innervated by a single nerve have EMG findings that are normal?
root A. Myasthenia gravis
E. A group of muscle fibers innervated by a single B. Polymyalgia rheumatica
peripheral nerve C. Amyotrophic lateral sclerosis (ALS)
D. Radiculopathy
. Which of the following is most likely an absolute contra— E. Carpal tunnel syndrome
indication to needle electromyography (EMG)?
A. Human immunodeficiency virus . EMG and nerve conduction studies provide all of the
B. Cardiac pacemaker following diagnoses EXCEPT:
C. Pregnancy A. Demyelinating neuropathy
D. Placing a needle through an infected site B. Axonal neuropathy
E. Severe thrombocytopenia C. Primary muscular neuropathy
D. Neuropathy caused by uncontrolled diabetes
What is the most common complication of needle elec- E. Total denervation injury
trode examination?
A. Infection . What is the main reason to perform an H—reflex study?
B. Bleeding A. Evaluate for an L4 radiculopathy
C. Bruising B. Evaluate for an L5 radiculopathy
D. Weakness C. Evaluate for an 81 radiculopathy
E. Soreness D. Evaluate for a C8 radiculopathy
E. None of the above
Which of the following disorders would most likely be
normal on nerve conduction studies (NCS) /EMG testing? 10. What is the most severe form of nerve injury and con-
A. Peripheral neuropathy sists of severe disruption or transection of the nerve?
B. Myopathy A. Neurotmesis
C. Radiculopathy B. Axonotmesis
D. Multiple sclerosis C. Neuropraxia
E. Amyotrophic lateral sclerosis D. Wallerian degeneration
E. Cryptogenic neuropathy
All of the following will affect nerve conduction velocity
EXCEPT: ll. What is the most sensitive diagnostic test for distal
A. Temperature of the limb small-fiber neuropathy?
B. Patient’s height A. Electromyography
C. Myelination B. Quantitative pseudomotor axon reflex test
D. Patient’s BMI C. Somatosensory evoked potentials
E. Patient’s age D. Nerve conduction studies
E. Motor evoked potentials
What is the distance between the two stimulus points
if the motor conduction velocity is 5 m/sec, the proxi- 12. Which of the findings on a needle examination would
mal motor latency is 2 msec, and distal motor latency is be typical of an axonal neuropathy?
l msec? A. 2—3-msec burst of electrical activity of 50—250
A. 5 cm mV on initial insertion of the needle into the
B. 10 cm muscle

28
CHAPTER 14 — Electromyography and Evoked Potentials 29

B. A “full” interference pattern on maximum voluntary B. The nerve conduction pathway spares the primary
effort sensory cortex.
C. Presence of an F—wave C. There is no accepted correlation between SEPs and
D. Positive sharp waves disorders affecting joint and position sense.
E. Polyphasic potentials with reduced amplitude D. SEPs cannot be accurately obtained by stimulation
of cranial nerves.
13. A patient presents to your clinic with right arm pain and E. SEPs are not useful in patients with peripheral
weakness. Her recent EMG/NCS report shows normal neuropathy.
sensory and motor nerve conduction studies. However,
on needle examination, she has active denervation noted 18. In the somatosensory evoked potential pathway, a stim-
in the cervical paraspinals, right fiexor carpi radialis, and ulus is propagated from the peripheral nerve to the
right brachioradialis. EMG of the triceps brachii and del- brain via which of the following routes?
toid were normal. What is the likely source of her pain? A. Dorsal root ganglion (DRG), contralateral dorsal
A. Right C5 radiculopathy column, medial lemniscus, thalamus, cortex
B. Right C6 radiculopathy B. DRG, ipsilateral dorsal column, thalamus, medial
C. Right C8 radiculopathy lemniscus, cortex
D. Right median neuropathy C. DRG, medial lemniscus, contralateral dorsal column,
E. Right radial neuropathy thalamus, cortex
D. DRG, ipsilateral dorsal column, medial lemniscus,
14. Which of the following is the “gold standard” for the thalamus, cortex
electrodiagnostic evaluation of radiculopathies? E. DRG, contralateral dorsal column, thalamus, medial
A. Somatosensory evoked potentials lemniscus, cortex
B. Motor evoked potentials
C. Event-related potentials 19. Which of the following SEP responses is thought to
D. Hoffman’s reflex reflect activity in the nerve roots of the cauda equina?
E. Electromyography A. LBS
B. T118
15. What is the most common visual evoked potential C. P37
abnormality seen with multiple sclerosis? D. N45
A. Prolonged N75 latency E. None of the above
B. Prolonged N145 latency
C. Prolonged P100 latency 20. In regards to motor evoked potentials (MEPs), the
D. Decreased P100 amplitude following is TRUE:
E. Decreased N75 amplitude A. Electrical stimulation is more common than mag—
netic stimulation because of better accuracy of
16. Which of the following statements is FALSE concern- electrical over magnetic.
ing brainstem auditory evoked potentials (BAEPs)? B. The clinical utility of MEPs is in the diagnosis of
A. Abnormal BAEPs are demonstrated in Friedreich’s disorders that affect the central or peripheral
ataxia. motor pathways.
B. Testing is limited with marked hearing loss. C. The most accepted way of measuring MEPs is by
C. Abnormal BAEPs are demonstrated in patients with needle insertion.
multiple sclerosis. D. MEPs are generally not useful in evaluation of
D. They are extremely useful in the diagnosis of acous- parenchymal brain injury, such as in the case of a
tic neuromas. cerebrovascular accident.
E. Testing is limited in comatose patients or those E. All of the above
under general anesthesia.

17. In regards to somatosensory evoked potentials (SEPs),


the following is TRUE:
A. Recording of the SEPs response depends on stimu-
lation of large, fast-conducting sensory fibers in the
peripheral nerve.

ANSWERS
1. A. A motor unit is a group of muscle fibers innervated 2. D. Many sources report no absolute contraindications
by a single anterior horn cell. All of the motor units to EMG; however, the most likely absolute contraindica-
within a muscle are considered a motor pool. The junc- tion to needle EMG is placing a needle through an in-
tion between a single muscle fiber and a branch of a fected site. Relative contraindications include severe
motor neuron is the neuromuscular junction. thrombocytopenia and lymphedema. Cardiac devices
PART 3 — EVALUATION AND ASSESSMENT

such as pacemakers and implanted cardiac defibrillators neuron, and then returns down the motor fibers of
are relative contraindications for stimulation during the calf muscle. It may be redundant when a depressed
NCS. Pregnancy and HIV are not contraindications. ankle reflex has already been noted on physical exam.
Also, the opposite leg must be studied to show a nor-
. E. Complications from needle electrode examination mal H-reflex as a contrast because bilateral absent
are rare, but the most common is transient muscle sore- H—reflex may be due to peripheral neuropathy. Other
ness. Other potential complications include infection, pitfalls include the lack of a good H—reflex in many
bleeding/bruising, nerve injury, and pneumothorax. older patients and inability to tell when the injury
occurred if needle EMG is normal.
. D. Findings on EMG are almost always normal in dis-
eases of the central nervous system, such as multiple 10. A. Nerve injury includes, from mildest to most severe,
sclerosis. While nerve conduction studies are often nor- neuropraxia, axonotmesis, and neurotmesis. Neurot-
mal in radiculopathy, diagnosis is established using nee- mesis is a severe disruption or transection of the nerve.
dle EMG. NCS/EMG can be used to help diagnose a In this type of injury, neuromas may form, which
number of conditions including peripheral neuropathy, may be painful, and surgical reanastomosis may be re-
myopathy, and amyotrophic lateral sclerosis. quired. Neuropraxia is a loss of conduction without
associated changes in axonal structure. Axonotmesis is
. D. Of the factors listed, temperature of the limb, patient’s nerve injury where the axon is disrupted in its myelin
height, myelination, and a patient’s age will affect nerve sheath. It is also the type of nerve injury associated
conduction velocity. The patient’s BMI does not have an with Wallerian degeneration.
effect on nerve conduction velocity. Nerve conduction
velocity is increased with increased limb temperature, ll. B. Quantitative pseudomotor axon reflex test
decreased height, and with myelinated fibers. Velocity is (QSART) is the most sensitive diagnostic test for distal
decreased in infants and older adults. Although not an small-fiber neuropathy. It is a quantitative thermoregu—
option in this question, nerve conduction velocity is latory sweat test that is used to detect postganglionic
also affected by the diameter of the fiber—small fibers pseudomotor failure in neuropathies and pregangli—
are slower than large fibers. onic neuropathies with presumed transsynaptic
degeneration.
. C. Conduction velocity is based on the difference in
latencies between the two points of stimulation and the 12. D. A 2—8 msec burst of electrical activity of 50—250 mV
distance between the two points of stimulation. It is in amplitude on insertion of the needle into the
calculated by the following formula: muscle is normal insertional activity on needle exami-
MCV (m/sec) = DMM/(PML — DML) nation. A “full” interference pattern on maximum
DMM — distance between two stimulus points in voluntary effort is the interference pattern in normal
millimeters muscle. The F—wave is a small, late muscle response
PML — proximal motor latency in milliseconds that occurs from backfiring of anterior horn cells. It
DML — distal motor latency in milliseconds is used for suspected early Guillain-Barré syndrome.
MCV — motor conduction velocity in m/ second Potentials that are reduced in amplitude and polypha—
Solving for the distance between two stimulus points sic are seen in myopathies. Positive sharp waves are
leads to: spontaneous activity seen in denervated muscle from
DMM = MCV (m/sec) >< (PML — DML) axonal injury.
DMM = 5 m/sec X (2 msec — l msec) = 5 mm
13. B. First, motor and sensory nerve conduction studies
. B. While polymyalgia rheumatica is an important cause are rarely useful in radiculopathies. This is because of
of muscle pain and weakness, it is not a true myopathy. the fact that the lesion is proximal to the dorsal root
Strength is maintained, and inflammation is at the level ganglion. This is why needle examination is needed
of the synovium and bursa. EMG findings are also nor- when evaluating a patient for radiculopathy. This pa-
mal in patients with this condition. All the other condi- tient has active denervation in the cervical paraspinals,
tions will most likely have abnormal findings on EMG. so we know this likely involves a nerve root instead of a
peripheral nerve. There is also active denervation in
. D. EMG and NCS are useful for localizing sites of neu- the right flexor carpi radialis (C6, C7, C8) and right
romuscular disorders and providing information about brachioradialis (C6, C7). However, EMG of the triceps
the nature of the process, whether primarily demyelin- brachii (C7, C8) and deltoid (C5, C6) were normal,
ating, axonal, primary muscular, or radicular. Unfortu— which makes a C5 and C8 radiculopathy less likely.
nately, it cannot provide the cause of the process or
diagnosis (diabetes, tumor, Guillain-Barré syndrome). 14. E. Electromyography is the gold standard for the elec-
trodiagnostic evaluation of radiculopathies. Somato—
. C. The H-reflex, or H-reflex latency, is carried mostly in sensory evoked potentials have been employed by us-
the 81 nerve root distribution; therefore, the main rea- ing “segmentally specific” techniques and spinal SEPs
son for the study is to evaluate for an 81 radiculopathy. rather than cortical SEPs, but they are controversial.
The stimulation impulse travels up the sensory fibers of They seem to be most useful for radiculopathies where
the posterior tibial nerve, synapses with the alpha motor sensory symptoms predominate.
CHAPTER 14 — Electromyogrophy and Evoked Potentials 31

15. C. Visual evoked potentials evaluate pathology affect- usually normal when only pain and temperature sensa-
ing the visual pathways and are used primarily for the tion is affected.
diagnosis of multiple sclerosis. The VEP response
consists of three peaks—N75, P100, and N145 (N for 18 D. The somatosensory evoked potential pathway
negative, P for positive, and the number indicating la- starts with stimulation of large, fast-conducting sen-
tency in msec). N75 and N145 are of less clinical rele— sory fibers in the peripheral nerve. The impulse then
vance. The VEP abnormality most commonly seen with enters the spinal cord through the dorsal root gan-
multiple sclerosis is prolongation of P100 latency. glion and ascends in the ipsilateral dorsal columns.
It then crosses to the contralateral side at the medial
16. E. Brainstem auditory evoked potentials assess the lemniscus and continues to the contralateral ventro-
auditory pathway from the middle ear structures posterolateral nucleus. It then proceeds to the pri—
through the eighth cranial nerve and brainstem and mary sensory cortex.
into the auditory cortex. They are extremely useful for
the diagnosis of acoustic neuromas and can also be 19 . A. When the tibial nerve is stimulated at the ankle
used to help diagnose multiple sclerosis and Friedreich’s during somatosensory evoked potentials testing, four
ataxia. The status of a patient does not usually affect peaks are expected—LBS (negative with 19 msec
the ability to obtain a BAEP response, including pa- latency), T118 (negative with 21 msec latency), P37
tients under general anesthesia and those that are (positive with 37 msec latency), and N45 (negative
comatose. Marked hearing loss can limit testing by with 45 msec latency). The L38 peak is thought to
degradation of the response. reflect activity in the nerve roots of the cauda equina
(T11 response from the dorsal fibers of the spinal
17. A. Recording of the SEPs response depends on stimu— cord, P37 and N45 from thalamocortical activity).
lation of large, fast-conducting sensory fibers in the pe-
ripheral nerve, and the pathway goes from the periph- 20. B. Motor evoked potentials are an extension of con-
eral nerve to the primary sensory cortex. In peripheral ventional NCS by allowing assessment of motor path-
nerve disease, cortical SEPs can be recorded when ways in peripheral neuronal structures and central
sensory nerve responses are not recordable by other motor conduction pathways. Stimulation is applied
techniques because of central nervous system amplifi- centrally and recorded peripherally. Electrical or mag-
cation. Conduction velocity can be calculated by stimu- netic stimulation can be used, but magnetic stimula-
lating two sites along the peripheral nerve and sub- tion is preferred because it is relatively painless.
tracting the cortical SEPs latencies. SEPs can also be Because it assesses the central motor conduction path-
used to identify trigeminal nerve lesions in multiple ways, it can be used to evaluate CNS pathology such as
sclerosis, and parasellar and cerebellopontine angle multiple sclerosis, Parkinson’s disease, cerebrovascular
tumors affecting the trigeminal nerve. SEPS can be accident, myelopathy in the cervical and lumbar spine,
correlated with proprioceptive disorders, but are plexus lesions, and motor neuron disorders.
5 Radiologic Assessment of the
Patient With Spine Pain

QUESTIONS
1. In the primary care setting, which of the following is B. A three-view lumbar spine x—ray is equivalent in radia-
the most common systemic pathologic process causing tion exposure to five chest x—rays.
low back pain? C. A cervical X-ray is equivalent in radiation exposure to
A. Metastatic neoplasm two chest x—rays.
B. Osteoporotic compression fracture D. A lumbar computed tomography (CT) scan provides 6
C. Discitis mSv of exposure, a value equivalent to 60 chest x—rays.
D. Inflammatory spondyloarthropathy E. A technetium bone scan has a radiation dose of 6.3
E. Epidural abscess mSv or 63 chest x—rays.

2. Which of the following is true about imaging of asymp- . Which of the following is the strongest red flag predictor
tomatic patients? of a neoplastic process underlying a patient’s spine pain?
A. Up to 95% of patients above the age of 50 have age- A. Unexplained weight loss
related disk changes on lumbar x—ray (disk space nar- B. Age > 70
rowing, anterior osteophytes, vacuum phenomena). C. History of cancer
B. Approximately 60% of patients older than 60 years D. Immunosuppression
undergoing lumbar magnetic resonance imaging E. Progressive neurologic deficit
(MRI) have abnormalities that would be considered
significant in an appropriate clinical setting. . What type of Modic change is seen at the top of the L5
C. In young adults age 20—22 years, lumbar MRI reveals vertebral body in Fig. 15.1?
grade 3 or higher Pfirrmann disk degeneration in A. Type I
approximately half of individuals. B. Type II
D. In healthy adults with mean age 40, T2 signal loss C. Type III
and disk herniations are most common at the L4 and D. None of the above
L5 segmental levels (25%—35% of disks) with a high E. Both A and B
prevalence of asymptomatic high intensity zones
(24% at L4 and L5).
E. All of the above

. Position and loading play an important role in spinal


anatomy and affect imaging sensitivity. All of the follow—
ing are true EXCEPT:
A. Lumbar extension reduces cross-sectional area of the
central canal.
B. Lumbar flexion decreases neuroforaminal cross-
sectional area.
C. Cervical extension reduces cross-sectional area of the
central canal.
D. Cervical flexion reduces cross—sectional area of the
central canal, but less so than extension.
E. Cervical flexion increases neuroforaminal cross-
sectional area.

. Radiation exposure from spinal imaging is a significant


health concern. Which of the following statements is
FALSE?
A. A conventional chest x—ray provides 1/30 (0.1 mSv) Fig. 15.1 Reprinted from Benzon H, Rathmell JP, Wu CL, et al. Prac-
of the annual natural background radiation expo- tical Management of Pain. 5th ed. Philadelphia, PA: Elsevier Mosby;
sure in North America (3 mSv). 2014:200.
CHAPTER 15 — Rodiologic Assessment of the Patient With Spine Pain 33

7. Which of the following statements is most accurate


regarding imaging correlates of discogenic pain as
predicted by positive diskography?
A. Loss of nuclear disk signal is thought to be most
predictive.
B. Severity of disk height loss is not predictive of disco-
genic pain.
C. High-intensity zones (HIZ) are always indicative of
painful disks.
D. Modic marrow endplate changes are less significant
when they involve >25% of the vertebral body.
E. Only Modic I changes have correlated with positive
diskography.

8. Which of the following is true regarding imaging of


zygapophyseal joints and facet joint pain?
A. Morphologic facet changes such as subchondral
sclerosis, erosions, cysts, osteophytes, or joint space
narrowing are nonspecific age-related changes. Fig. 15.2 Reprinted from Benzon H, Rathmell JP, Wu CL, et al. Prac-
B. There is no association between MRI findings of tical Management of Pain. 5th ed. Philadelphia, PA: Elsevier Mosby;
structural facet arthrosis and pain relief with dual 2014210.

medial branch blocks.


C. T2 hyperintensity within and around the facet
joint including the adjacent pedicle and multifidus 11. Pain in Bertolotti’s syndrome may emanate from which
muscle has been associated with axial pain of facet structure?
origin. A. Transitional segment neoarticulation
D. Gadolinium enhancement of the facet joint is sug- B. Contralateral facet joint at the level of an asymmet—
gestive of facet synovitis and possibly pain of facet ric neoarticulation
origin. C. Discogenic pain from the disk above the transitional
E. All of the above segment
D. Radicular pain from herniation in the superior
9. Which of the following is FALSE regarding imaging of adjacent segment disk
the sacroiliac (SI) joint and SI joint pain? E. All of the above
A. The inferior and anterosuperior aspects of the
radiographically visualized joint are synovial. 12. A patient presents with radiculopathy, and the follow—
B. Although the surface area of the joint is large, ing MRI image is obtained (Fig. 15.3). Using the ap-
its synovial space volume is small ranging from propriate nomenclature, describe the L5—S1 herniation
1—2.5 cc. and the anticipated level of radiculopathy.
C. Radiographic narrowing of the SI joint, subchon- A. Focal protrusion, left L5 radiculopathy
dral sclerosis, osteophyte formation, and vacuum B. Focal protrusion, left S1 radiculopathy
phenomena within the joint indicate that the C. Extrusion, left L5 radiculopathy
patient has SI joint pain. D. Extrusion, left SI radiculopathy
D. Technetium bone scans are more specific than E. Sequestration, left L5 radiculopathy
radiography for identifying patients with SIjoint
pain as referenced by intraarticular injections.
E. MRI evidence of active SI joint inflammation with
gadolinium enhancement has been correlated with
disease activity in sacroiliitis secondary to spondylo-
arthropathy.

10. A 75-year-old patient presents with focal midline


lumbar pain and intermittent bilateral leg pain,
worse with extension and ambulation. Imaging with
MRI (Fig. 15.2) reveals a posterior midline epidural
cyst with neural compression. There is apposition of
the L4 and L5 spinous processes. The most likely
etiology is:
A. Bertolotti’s syndrome
B. Baastrup’s disease
C. Zygapophyseal joint arthropathy Fig. 15.3 Reprinted from Benzon H, Rathmell JP, Wu CL, et al. Prac-
D. Epidural metastasis tical Management of Pain. 5th ed. Philadelphia, PA: Elsevier Mosby;
E. Epidural abscess 2014:216.
34 PART 3 — EVALUATION AND ASSESSMENT

13. What is the best imaging modality for distinguishing B. Facet hypertrophy
postoperative epidural scarring from recurrent disk C. Ligamentum flavum hypertrophy
herniation? D. Short pedicles
A. CT myelography E. Spondylolisthesis
B. MRI without contrast
C. MRI with contrast
D. Bone scan
E. SPECT/CT

14. Which of the following is FALSE regarding cervical


radicular pain and disk herniations?
A. The C5 and C6 nerve roots are most commonly
affected.
B. Cervical radicular pain is typically caused by a multi—
factorial etiology including uncovertebral and facet
joint hypertrophy, loss of disk height, and, less com—
monly, disk herniation.
C. The vector of cervical disk herniation is typically
posterolateral.
D. Since cervical nerves exit low in the foramen, a
herniation will likely affect the exiting nerve rather
than the traversing nerve.
E. Since most cervical radiculopathy etiologies are at
least in part bony, complete resolution of pain is
less likely than in the lumbar region where a purely Fig. 15.4 Reprinted from Benzon H, Rathmell JP, Wu CL, et al. Prac-
disk etiology is more common. tical Management of Pain. 5th ed. Philadelphia, PA: Elsevier Mosby;
20142225.
15. What is the dominant structural cause of this patient’s
lumbar spinal stenosis (Fig. 15.4)?
A. Disk bulge

ANSWERS

1. B. Osteoporotic compression fractures are the most 6. A. Type I Modic change exhibits hypointense (dark)
common systemic pathologic process underlying low signal on T1 and hyperintense (bright) signal on T2
back pain in the primary care setting (4% of patients). MRI, and may enhance with gadolinium. It is indica—
Only 0.7% suffer from undiagnosed metastatic neo- tive of vascularized granulation tissue.
plasm. Spine infection (diskitis, epidural abscess) is
present in only 0.01% of patients. Inflammatory spondy- A. The rank correlation of MRI findings with a positive
loarthropathies, such as ankylosing spondylitis, account diskogram was as follows: signal abnormality > disk
for 0.3% of presentations. height > disk contour > HIZ > end plate change.
Disk signal change alone was as accurate as other
. E. The evidence is robust that a significant percentage individual parameters or combinations.
of asymptomatic patients present with age-related changes
on imaging that are nonspecific for pain. As such, corre- . E. Facet morphologic changes are nonspecific for pain
lation of imaging findings with pain presentation is and occur universally with age. Signs of edema, hyper-
essential. emia, and accelerated metabolic activity suggestive of
active inflammation, best detected by bone scan or
3. B. Lumbar flexion increases both neuroforaminal and SPECT/ CT as well as T2 signal, may be most predictive
central cross-sectional area. In the cervical spine, how- of facet-mediated pain.
ever, flexion somewhat reduces central canal area rela-
tive to the neutral position, but increases neuroforami- . C. Like the facet joint, age-related morphologic
nal area. changes of the SI joint are common above age 40 and
not predictive of SI joint pain.
4. B. A typical lumbar spine series (1.5 mSv) has radiation
exposure of 15 chest x—rays (0.1 mSv). 10. B. This is an interesting presentation of Baastrup’s dis—
ease, whereby the L4 and L5 spinous processes are in
. C. History of cancer is the strongest predictor of a neo- midline contact, resulting in formation of a pseudo-
plastic process as the underlying cause of new onset bursa that extends anteriorly through a midline cleft
spine pain. in the ligamentum flavum and presents as a midline
CHAPTER 15 — Rodiologic Assessment of the Patient With Spine Pain 35

posterior epidural cyst with claudication symptoms due Early postgadolinium images in the postoperative
to spinal stenosis. patient will show recurrent disk herniation as a nonen—
hancing zone; enhancing epidural fibrosis may sur-
ll. E. Bertolotti’s syndrome involves transitional lumbosa— round this.
cral segment—related pain, which may arise from mul-
tiple pain generators, as listed in the question stem. 14. A. The most commonly affected cervical nerve roots
from disk herniation are the C6 and C7 nerve roots.
12. D. This L5—Sl disk herniation is an extrusion by sagit— These herniations typically take place at the (15-6 and
tal imaging; its paracentral and subarticular location C6-7 levels, respectively.
will impact the traversing/ descending left 81 nerve
root, resulting in the left 81 radiculopathy described. 15. C. The ligamentum flavum (dark) is the dominant
structure causing this patient’s lumbar spinal stenosis.
13. C. MRI with gadolinium enhancement is very accurate While the facet joint does demonstrate hypertrophy, it
in distinguishing recurrent disk herniation from epi— is not the primary cause of this patient’s spinal narrow—
dural fibrosis. Epidural fibrosis enhances rapidly and ing. Short pedicles and spondylolisthesis cannot be
uniformly following gadolinium administration; disk identified from this axial MRI slice. The disk shows no
material does not enhance for the first 20—30 minutes. posterior herniation or bulge.
II 6 Psychological and Behavioral
Assessment

QUESTIONS

1. Which of the following domains is assessed by the C. Patient Outcome Questionnaire


Patient Outcome Questionnaire? D. McGill Pain Questionnaire
A. Physical functioning E. West Haven-Yale Multidimensional Pain Inventory
B. Pain beliefs and coping
C. Quality of care . During biofeedback treatments, which of the following
D. Personality assessments is most commonly employed?
E. Emotional functioning A. Psychophysical measures
B. Questionnaires and inventories
2. According to the Initiative on Methods, Measurement, C. Behavioral observations
and Pain Assessment in Clinical Trials (IMMPACT) D. Diaries
committee’s review, which pain intensity scale is recom- E. Reports from family and significant others
mended for chronic pain clinical trials in adults?
A. Numerical Rating Scale (NRS) 6. The IMMPACT group encouraged the use of measures
B. Verbal Rating Scale (VRS) that were developed and normed for a specific popula-
C. Visual Analog Scale (VAS) tion (e.g., osteoarthritis). Which of the following is a
D. FACES Pain Scale disease-specific measure?
E. Color Analog Scale (CAS) A. Pain Catastrophizing Scale
B. Pain Disability Index
Psychological and behavioral assessment can be used C. Beck Depression Inventory
for all of the following goals EXCEPT: D. Oswestry Disability Questionnaire
A. Provide multidimensional information regarding the E. McGill Pain Questionnaire
patient’s pain
B. Determine if the patient is malingering 7. Which inventory primarily measures physical function?
C. Determine if the patient requires adjunctive psycho- A. West Haven-Yale Multidimensional Pain Inventory
logical treatment B. Beck Depression Inventory
D. Determine the patient’s baseline pain complaint and C. Brief Pain Inventory
functioning D. Patient Outcome Questionnaire
E. Provide information about a patient’s suitability for E. Patient-Reported Outcomes Measurement Informa-
invasive procedures tion System

4. One of your patients with chronic pain is given an assess-


ment that classified him as “interpersonally distressed.”
Which assessment was he mostly likely given?
A. Minnesota Multiphasic Personality Inventory
B. Survey of Pain Attitudes

ANSWERS

1. C. The Patient Outcome Questionnaire (POQ) was pro- 2. A. All of the choices are valid measures of pain inten—
duced by the American Pain Society to assess the quality sity; however, the IMMPACT group recommended the
of patient care, specifically the patient’s perceived quality 11-point numerical rating scale (NRS) to assess pain
of pain treatment. The questionnaire consists of 12 items intensity for chronic pain clinical trials. The main rea—
that assess pain severity; the impact of pain on affect, sons were that respondents may prefer VRS and NRS
sleep, and activity; side effects; satisfaction with informa- over the VAS, the VAS instruments are more likely to
tion provided about the pain treatment; shared decision result in missing or incomplete data, and respondents
making; and use of nonpharmacologic strategies. with limited vocabulary or command of English may

36
Another random document with
no related content on Scribd:
LITTLE EARL STREET, LOOKING
EAST

PLATE 42

Nos. 14 to 16, NEW COMPTON


STREET, SHOP FRONTS
PLATE 43

PLATE 44
PLATE 45
St. GILES-IN-THE-FIELDS. WEST
FRONT.

PLATE 46
St. GILES-IN-THE-FIELDS. CROSS
SECTION.

PLATE 47
ST. GILES-IN-THE-FIELDS CHURCH
FROM THE NORTH-WEST

PLATE 48
ST. GILES-IN-THE-FIELDS CHURCH
FROM THE NORTH-EAST

PLATE 49
ST. GILES-IN-THE-FIELDS CHURCH, INTERIOR, LOOKING
EAST, 1753

PLATE 50
ST. GILES-IN-THE-FIELDS CHURCH, INTERIOR, LOOKING
WEST

PLATE 51
ST. GILES-IN-THE-FIELDS CHURCH,
COLUMNS AND CEILING

ALTARPIECE
PLATE 52

ST. GILES-IN-THE-FIELDS CHURCH,


OAK FRAME WITH PICTURE
PAINTED GLASS PANEL, PROBABLY
FROM FORMER CHURCH

PLATE 53
PLATE 54
ST. GILES-IN-THE-FIELDS CHURCH, OAK PANEL IN LICH
GATE

PLATE 55
ST. GILES-IN-THE-FIELDS CHURCH, VESTRY

PLATE 56
PLATE 57
PLATE 58
PLATE 59
Nos. 10 and 11, DENMARK STREET

PLATE 60
DENMARK PASSAGE, BLACKSMITH’S FORGE

PLATE 61

You might also like